Which psychological theory best explains the role of community values in the preliminary studies of tobacco cessation?

Section Bank: Psychological, Social, and Biological Foundations of Behavior: Passage 1

1) To answer this question, we can jump back to the passage and do a quick recap of Phase 1 and what the participants were asked to do. Something I want you to note: I’m not going through the answer choices just yet. I’m going to try and break down the question using the passage and my general knowledge of the behavioral content. Why is that? I don’t want the answers making me biased as I go through them. This way, I’m not being influenced by the possible answers or trying to justify an answer even though it may be incorrect. 

Here we have the part of the passage that talks about phase 1. I want you to focus on what the participants were asked to do. It says “Participants were instructed to repeat immediately any digits they heard in the attended ear.” 

We can look at this within the context of the entire phase. The digits come in to the relevant ear, and the participants have to recall and repeat the digits they heard in the attended ear, and within the stream of bisyllabic words. The key here is what the participants have to do next. They’re asked to recall, and then immediately repeat the appropriate digits from the attended ear, to the researchers. Now in theory, we should be done. Hopefully there’s an answer choice that mentions that sentence or something along the lines of “recalling digits from attended ear and repeating those digits”. We’re going to go through the answer choices one by one, and methodically eliminate incorrect answer choices. 

  1. Shadow the attended ear. Right away, this answer choice sounds like our breakdown. What is shadowing? Essentially what happened in Phase 1. A participant in a study will repeat back words, as other stimuli are mixed in. In Phase 1, we had digits played in the ears of the participants. The participants had to recall and repeat the digits presented to the attended ear only. These digits were mixed in, within the stream of bisyllabic words. Textbook example of shadowing, so we’re going to hold on to answer choice A for now. But we’re not done. Remember, we always want the best answer choice. We might have an answer choice that’s more specific, or more correct in some way. 
  2. Shadow the unattended ear. Phase 1 didn’t involve shadowing the unattended ear. We don’t want the participants recalling and repeating the digits from the unattended ear, we can eliminate answer choice B because it contradicts what we read in the passage.
  3. Recall the information presented on the attended ear. This answer choice is interesting. The participants do have to recall some information presented on the attended ear, but they’re also asked to repeat specific information. In fact, the author explicitly says this. This answer choice is incomplete. It doesn’t tell the whole story like answer choice A.
  4. Repress the information presented on the unattended ear. This answer choice is again something that’s tangentially related to the correct answer. The participants don’t want to recall and repeat the information presented on the unattended ear. But what is this question asking? What are the participants instructed to do? They’re instructed to repeat immediately any digits they heard in the attended ear. While they may have to repress information presented on the unattended ear, that’s not what they’re explicitly instructed to do. Make sure to read the question carefully. We’re left with our correct answer, answer choice A.

2) The passage talked about there being more intrusion errors in certain circumstances, so we can quickly reference what the author says.

Which psychological theory best explains the role of community values in the preliminary studies of tobacco cessation?

Here we have an excerpt from the passage, and we’re concerned with the topic on our content outline titled: Brain Areas That Control Language and Speech. I also added a nice visual right below. 

The passage says Most intrusions of digits from the unattended ear occurred when participants were instructed to attend to the left ear. Intrusions are the opposite of omissions. An intrusion related to the right ear would mean one of the incorrect digits heard in the right ear was recalled and repeated. That’s despite the fact the participants were instructed to attend to the left ear. The digit from the unattended, right ear was not meant to be repeated, but it was mistakenly. 

We have to think about the input that comes into each ear, and travels to our brains. Sound is similar to vision, in that input coming into the left side (or left ear in this case), is going to go to the right hemisphere, and vice versa. Is that going to cause any problems for us, or would that explain the discrepancy in intrusion errors? Well, think about it this way: think big picture and more generically. Which side of the brain is responsible for language and speech? It’s the left side, or left hemisphere. Look at the image right below the excerpt. If we have input traveling into the left ear, that input goes to the right hemisphere of the brain. That’s all well and good, but we said the left hemisphere is the side we associate with being responsible for language and speech. Looking at this from the other perspective: the input that goes into the right ear, then travels to the left hemisphere, will be processed in that same hemisphere. That should theoretically produce fewer errors. 

  1. Verbal inputs to the right ear, which first go to the auditory cortex in the right hemisphere, must be processed by the language areas of the left hemisphere. Right away, we see an issue. Where should verbal inputs to the right ear go? Auditory cortex in the opposite hemisphere, or the left hemisphere. Final part of the answer is good: input is processed by language areas of the left hemisphere. 
  2. Verbal inputs to the right ear, which first go to the auditory cortex in the left hemisphere, must be processed by the language areas of the right hemisphere. First part of this answer choice looks much better. We have verbal input to the right ear and the input goes to the auditory cortex in the left hemisphere. But that’s also where it should be processed. This answer choice incorrectly says the input must be processed in the right hemisphere. Both A and B aren’t great, but both are only partially incorrect. 
  3. Verbal inputs to the left ear, which first go to the auditory cortex in the left hemisphere, must be processed by the language areas of the right hemisphere. This answer choice combines the incorrect parts of A and B. The input to the left ear should go to the auditory cortex in the right hemisphere. It’s processed by language areas of the left hemisphere.
  4. Verbal inputs to the left ear, which first go to the auditory cortex in the right hemisphere, must be processed by the language areas of the left hemisphere. This answer choice sounds like a keeper. We have verbal input to the left ear. That goes to the auditory cortex in the right hemisphere and must be processed by the language areas of the left hemisphere. That difference is what ultimately causes more intrusion errors. This is the best answer choice of the choices listed, so we’re left with our correct answer, answer choice D: Verbal inputs to the left ear, which first go to the auditory cortex in the right hemisphere, must be processed by the language areas of the left hemisphere

3) As we read through the passage, we focused on some correlations, and we want to be careful to pick an answer choice that is NOT suggested by those correlations. Be careful with the verbiage here. Ultimately, this comes down to eliminating 3 answer choices that’re consistent with the passage. Our correct answer is going to be the odd-one-out.

We have an excerpt from the passage, and the last bit here says There was a strong negative correlation between the number of errors in Phase 2 and the successful completion of pilot training. Then last sentence here says The number of errors in Phase 2 had low correlations with all other tests used for pilot selection. There it is! We have our correlations. For the first sentence a high number of errors correlates to a low success rate in pilot training. A low number of errors correlates to a higher success rate in pilot training. Negative correlation.

For the last sentence, we have a low correlation. A low correlation isn’t the same as a negative correlation. A low correlation just means there isn’t as clear of a relationship between the variables. 

To summarize our correlations: the number of errors in Phase 2 correlated to whether the pilot completes the training or not. The number of errors does not correlate with other tests used for pilot selection. 

  1. the study’s measure of attention capabilities enhances the predictive validity of the pilot selection test battery. And the pilot selection test battery is just referencing the selection process for pilots. Phase 2 was a test of maintaining and reorienting attention. The number of errors in phase 2 correlated to whether the pilot completes the training or not. That means the correlations do suggest answer choice A. Be careful here, the question stem says “EXCEPT” in big, bold letters. Never have that be the reason you miss a question. 
  2. the study’s measure of attention capabilities reduces the predictive validity of the pilot selection test battery. This answer choice sounds like the opposite of answer choice A, and the opposite of our breakdown. We said phase 2 was a test of maintaining and reorienting attention. Errors in phase 2 correlated to whether the pilot completes the training or not. That means it enhances the predictive validity. We can keep answer choice B for now, but we can eliminate answer choice A-it contradicts what we were looking for in the question stem, because of what we read in the passage.
  3. flight cadets who perform better on the task are more likely to complete training on more advanced equipment than those whose performance is poorer. This answer just sounds like answer choice A. The cadets that perform better on the task, or the ones who had the best attention capabilities are most likely to complete training.  We can also eliminate answer choice C for contradicting the question stem, because of what we read in the passage.
  4. flight cadets who make more errors in Phase 2 are less likely to complete training on more advanced equipment. Another answer choice that’s consistent with what we read in the passage. Biggest issue here would be misreading the question and not seeing the question stem asks for something that’s not a correlation. We’re left with our correct answer, answer choice B.

4) To answer this question, we have to ask: what can measuring the ability to redirect attention tell us about accident rates in bus drivers? We can focus on Phase 2 because we’re focused on redirecting attention, which is something we just focused on in our last question. That was what the researchers looked at in Phase 2. Errors in phase 2 were obviously bad. We said a high number of errors correlated to not completing pilot training. The opposite is also true. A lower number of errors correlated to successful completion of pilot training.

  1. Phase 1 will be directly related to accident rates. This answer choice isn’t as relevant to the question being asked. Phase 1 was not the phase that focused on the ability to redirect attention. That was phase 2. Why is redirecting attention so important? Think about how much bus drivers (and drivers in general) have to redirect their attention. There are constantly cars going around them, next to them, sometimes toward them. Redirecting attention is going to be vital to not crashing. 
  2. Phase 1 will be inversely related to accident rates. Same issue as answer choice A. Now that being said, if we have a higher rate in general, that likely means an inverse relationship to accident rates. We can eliminate answer choice A for now, but we still want an answer choice that focuses on Phase 2. 
  3. Phase 2 will be directly related to accident rates. This answer choice sounds pretty solid at first glance. We said Phase 2 focuses on the ability to redirect attention. Error rates in Phase 2, should therefore be directly related to accident rates. Not maintaining and reorienting attention is a bad thing for pilots and bus drivers. That would directly relate to accident rates. This is our best answer so far.
  4. Phase 2 will be inversely related to accident rates. This answer choice is the opposite of answer choice C, so right away we don’t like it. This is a better option than answer choice A, but being in the top 3 isn’t a good enough reason to pick this answer. It still contradicts what we know from the passage. We’re left with our correct answer, answer choice C: Phase 2 will be directly related to accident rates

5) To answer this question, we can go back to what we read in the passage. The purpose of the experiment is to develop or find a standardized test that can identify cadets that will be most proficient in flying. They want to weed out the bad candidates, and just be left with the best prospects. Let’s think back: Phase 1 asked participants to shadow their attended ear, and Phase 2 involved maintaining attention or reorienting attention. Ultimately, the task found a strong, negative correlation between the number of errors in Phase 2 and completion of the training. Said differently, the task weeded out the worst candidates.

  1. Yes, because cadets who completed training on more advanced equipment had performed significantly better on the task than those who dropped out of the training earlier. This sounds consistent with our breakdown. The ones that did poorly during the task are also the ones that dropped out of the training. The inverse is also true. Cadets that completed training were not weeded out during the task. 
  2. No, because performance in Phase 2 of the task did not predict performance on other tests used in pilot selection. While this may be true in a vacuum, we’re only focused on the main purpose here. The task was meant to identify cadets that are most proficient at flying by weeding out bad candidates. We can eliminate this answer choice, it’s not addressing the specific question being asked. 
  3. Yes, because cadets’ ability to divide attention between simultaneous inputs was predicted by their performance in Phase 1. This is similar to answer choice B. The best correlation for our purpose was being able to redirect attention in Phase 2, not dividing attention in Phase 1. We can eliminate this answer choice for the same reason. 
  4. No, because cadets’ performance in Phase 1 did not predict their ability to redirect attention in Phase 2. This answer choice is also a bit out of scope. Even if Phase 1 didn’t predict ability in Phase 2, the main correlation came from the results in Phase 2. Sure, having good results in both phases would be great, but that’s not what we’re looking for to answer this specific question. We can eliminate answer choice D, we’re left with our correct answer, answer choice A: Yes, because cadets who completed training on more advanced equipment had performed significantly better on the task than those who dropped out of the training earlier.

6) Said differently, what cognitive process was used in Phase 1? This is almost like a pseudo discrete, so make sure you know your vocab for this section! We know the flight cadets were required to repeat certain digits in Phase 1. Question 1 in this set was actually about this process also. We said the cadets were required to shadow the attended ear and only recall and repeat specific digits. They were expected to ignore any digits in the non-attended ear, and any extra words between the digits. 

  1. Divided attention. Divided attention is the ability of the brain to attend to two different stimuli at the same time. That’s not what we want here. We want to ignore the digits in the other ear, and ignore the bisyllabic words as well. If anything, we’d want selective attention.
  2. Sensory memory. This involves remembering sensory stimuli. In this case, the flight cadets weren’t concerned with memory, they were repeating the digits back almost instantly. Neither A, nor B are great, but neither stands out either. 
  3. Selective attention. Selective attention is demonstrated when many stimuli are present, and a person ignores the non-task-related stimuli. Sound like Phase 1? It sounds exactly like what the flight cadets had to do. They were focused on a specific set of digits while trying to tune out any words, and any digits in the non-attended ear. This describes what we read in the passage, we can now eliminate answer choices A and B. Neither one correctly identified the cognitive process. Again, this comes down to knowing your vocab.
  4. Sensory coding. This involves processing information through reception, transduction, coding, and awareness. This is outside the scope of this question and Phase 1. When the cadets were listening for the digits in their attended ear, that can best be described as selective attention. We can eliminate answer choice D so we’re left with our correct answer, answer choice C: Selective attention. 

Section Bank: Psychological, Social, and Biological Foundations of Behavior: Passage 2

7) This answer is going to come from going back to table 1 in the passage. There was a lot of data, so we can theoretically make an infinite number of observations, so we’ll look at the data while going through the 4 options presented to us.

  1. led to higher impulsivity scores during distraction tasks. Looking at Table 1, we’re looking at the impulsivity measurements. We have a vigilance task and we have a distractibility task related to impulsivity. This answer choice is talking about the distractibility score specifically. In the cocaine exposure group, we do have higher impulsivity scores, so this answer choice is consistent with the data in Table 1. Let’s hold on to this answer choice.
  2. was associated with poor achievement scores, especially for reading and science. We talked during our readthrough about the difference in math scores, but reading and science scores are actually similar. We can eliminate this answer choice.
  3. led to lower IQ scores than is typically found among children in the general population. IQ scores in Table 1 are similar, so nothing to indicate cocaine exposure led to lower IQ scores. We can eliminate this answer choice. 
  4. affected the intellectual and academic performance of low SES children more than that of their high SES peers. This may or may not be true, but we’re only dealing with low SES children. We don’t have numbers for their high SES peers, so we can eliminate this answer choice, we’re left with our correct answer, answer choice A: led to higher impulsivity scores during distraction tasks. 

8) In other words, why did researchers pick from such a specific group of people for their sample? We’ll have to think about why the researchers only picked women from the same city and socioeconomic status.

In research, you have variables. You can either control or measure these variables, and you can see the relationships between variables. When you’re researching something though, you have to think about any other variables that could affect your results. In this study, the researchers decided to minimize any outside factors like growing up in different cities or states. We also said in our readthrough, that by choosing only women that have low socioeconomic status, the researchers can try and ensure a similar background for the mothers. This allows the researchers to focus on their variables of interest.

  1. eliminated all the variables that would have confounded interpretation of the results. Confounding variables are extra variables that aren’t accounted for, but can alter your results. Ideally you can eliminate every confounding variable, but that’s usually not plausible. Like in this study, for example. A woman might not want to admit to cocaine use, so she changes her answers. By picking from a group of people with a specific background, researchers can avoid some confounding variables, but this answer is extreme. 
  2. set up an experiment to assess how cocaine might affect the human brain. For the research to be applicable to the human brain in general, it would have to apply to the entire world. In this case, the study was focused on children born to women having low socioeconomic status, and living in the same, large city. Limiting the sample didn’t allow researchers to assess how cocaine might affect the human brain in general.
  3. reduced variability due to differences in the quality of public schools. This is a viable answer choice. We said all of the women came from a large US city, and all had low socioeconomic status. This essentially makes it so most of the participants will have a similar background and environment. That similar environment means going to the same school, or at least being in the same district and quality of school. I’m liking this answer choice as our best option so far. A was too extreme, and B was unreasonable.
  4. were unable to draw any conclusions regarding prenatal cocaine exposure. This answer choice is also unreasonable. The researchers did draw conclusions in the passage. In fact, a majority of the passage revolved around their conclusions. This contradicts what we read in the passage. We’re left with our correct answer, answer choice C.

9) The passage mentioned measuring IQ scores of the children at age 6, but this is a pseudo-discrete question. We’re going to recall what we know about IQ scores and their distribution in a population. IQ is used to quantitatively measure intelligence. Average IQ is 100 and standard deviation is 15 points. We also know the majority of the population falls within one standard deviation of the mean. Meaning 68% of the population falls within that 85 to 115 range. Even with newer IQ tests, that score scale and breakdown remains roughly the same.

  1. more than two-thirds of children will score between 85 and 115. This answer choice matches our breakdown. We actually quantified this and said roughly 68% of the population will fall in a score range between 85 and 115. I’m liking this answer choice for now. 
  2. a quarter of all children will perform poorly on least one component of the exam. This is not an assumption of IQ-testing. Biggest defining characteristics are average IQ of 100, and standard deviation of 15 points. Scores are normalized to 100, and we don’t expect such a significant portion to perform terrible on any specific component of the exam. 
  3. significantly more children will score at the extremes of the scoring distribution than in the middle. This is the opposite of what we expect. Scores are normalized to the mean score of 100. We expect more children to score around this average than to score at the extremes. This contradicts what we know about IQ. 
  4. only exceptionally gifted children would have IQ scores above 85 by age 6; perfect test scores are rare during early childhood. This ties into what we saw earlier where scores are normalized to 100. Average IQ is 100, so it would be more than just the exceptionally gifted children with IQ scores above 85. We can eliminate this answer choice, it also contradicts our definition. We’re left with our correct answer, answer choice A: more than two-thirds of children will score between 85 and 115.

10) To answer this question, we want to pick the brain area least involved in the abilities from Paragraph 1. 

The behavioral section tests biology concepts as they relate to mental processes and behavior. That’s exactly what we’re looking for here. 

Numbers 1-3 all involve different issues that aren’t necessarily abilities. Number 4 gets into actual abilities: discrimination learning, and attention skills. We want the brain area least involved in learning and attention abilities. That means we can also rule out brain areas that deal with memory like the hippocampus, neo-cortex, and amygdala.  Those are related to these abilities.

  1. Frontal lobe. Frontal lobe deals with learning and attention skills. It’s involved in problem solving, memory, and judgment. 
  2. Hippocampus. Hippocampus deals with memory, and specifically long-term memory. Memory affects discrimination learning skills. 
  3. Hypothalamus. Hypothalamus has to do with controlling the autonomic nervous system, which means regulating bodily functions. It controls things like hunger, thirst, sleep, and regulates body temperature and secretion of hormones. It would be least involved in learning and attention abilities. 
  4. Thalamus. The thalamus serves as a relay station for information that comes and goes to the cortex. It plays a role in memory, which contradicts what we’re looking for in this question. We can also eliminate answer choice D. We’re left with our correct answer, answer choice C: Hypothalamus.

11) This question implies we’ll be given 4 experimental observations and we want one that supports the hypothesis that when a pregnant primate is administered cocaine, cocaine is pharmacologically active in the fetal brain. We’ll have to know some general information about cocaine, and how it’s typically metabolized and seen in the body. Cocaine is a stimulant that rapidly enters the bloodstream and penetrates the brain. It ultimately causes a buildup of dopamine. That’s the high people get; people experience increased pleasure and motivation. We know this elevated level of dopamine buildup is problematic. But for this question, we’re only focused on the mechanism. Let’s break down the 4 experimental observations.

  1. Glucose metabolism in the fetal brain increased. Why would glucose metabolism in the brain increase? Because there’s brain activity, and because the major source of energy for the brain is glucose. Cocaine is a stimulant, and stimulants lead to activity in the fetal brain. Increased activity in the brain would cause increased glucose consumption, like we said. We like this answer choice for now.
  2. Imaging studies showed that cocaine entered the fetal circulation. Cocaine entering the fetal circulation would need to happen for it to eventually be active in the fetal brain. But we don’t know if it actually crosses the blood-brain barrier, or if it’s active. The presence alone doesn’t mean it’s active in the fetal brain. Too many connections are made here that aren’t necessarily true. 
  3. The fetus had an increase in tolerance to pain. This answer choice is a stretch. Stimulants aren’t known to make people more tolerant to pain. We also don’t know if we can measure the difference in pain felt by the fetus, or if there’s any pain felt at all. Another answer choice where we’re making connections that aren’t necessarily true. 
  4. Imaging studies showed increased internalization of dopamine receptors. Internalization is usually synonymous with endocytosis. Internalization of dopamine receptors implies movement of receptors from the plasma membrane to the inside of the cell. Dopamine can’t bind to these receptors anymore, so we wouldn’t see the elevated effect of cocaine. We can eliminate this answer choice because it contradicts what the question stem is looking for. We’re left with our best answer, answer choice A: Glucose metabolism in the fetal brain increased

12) In other words, which answer choice describes a neurotransmitter? From the passage, we have to recall cocaine exposure during pregnancy alters the function of dopamine, serotonin, and norepinephrine (3 neurotransmitters). The author mentions this in the first sentence, but the answer is ultimately going to come from our general knowledge. This is like a pseudo-discrete question, or standalone question. We’re picking the characteristic that describe neurotransmitters.

  1. is only manufactured in endocrine glands throughout the body. This is false. Endocrine glands produce endocrine, autocrine, and paracrine molecules. Neurotransmitters are produced in neurons. This is factually incorrect.
  2. binds to a receptor on a postsynaptic membrane within the CNS. This describes a neurotransmitter. They bind to receptors on the postsynaptic membrane. This answer choice is just describing the mechanism by which they bind, so we like this answer choice. Answer choice A was factually incorrect, so we can eliminate that answer choice.
  3. is distributed to target neurons within the cerebrospinal fluid. Neurotransmitters are released at synapses of neurons. Synapses rely on the secretion of neurotransmitters across a synaptic cleft from the presynaptic neuron. The neurotransmitters will move over the synaptic cleft to the postsynaptic neuron. The neurotransmitters aren’t within the cerebrospinal fluid. This is factually incorrect. 
  4. transmits signals between two neurons at a faster speed than through gap junctions. Gap junctions are channels between neighboring cells that allow for transport. This can include ions, water, and other substances, but not neurotransmitters. We can eliminate this answer choice as well, it contradicts what we know about neurotransmitters. We’re left with our correct answer, answer choice B.

Section Bank: Psychological, Social, and Biological Foundations of Behavior: Questions 13-17

13) This is a standalone question which typically involves relying on external knowledge. There’s no associated passage, so the test-maker is testing how well you can read the question and utilize information from AAMC’s content outline. We can break down what we know about operant conditioning: it’s a type of associative learning process through which the strength of a behavior is modified by reinforcement or punishment. When we deal with operant conditioning, we think of shaping which involves a calculated reinforcement of a “target behavior”: it uses operant conditioning principles to train a subject by rewarding proper behavior and discouraging improper behavior. The method requires that the subject perform behaviors that at first merely resemble the target behavior; through reinforcement, these behaviors are gradually changed or “shaped” to encourage the target behavior itself.

Which psychological theory best explains the role of community values in the preliminary studies of tobacco cessation?

Note, in the context of operant conditioning, whether you are reinforcing or punishing a behavior, “positive” always means you are adding a stimulus (not necessarily a good one), and “negative” always means you are removing a stimulus (not necessarily a bad one). Let’s use this information and jump into our four options.

  1. The schedule unambiguously informs the subject which behavior is correct. This answer choice is consistent with operant conditioning in which the strength of a behavior is modified by reinforcement or punishment. The subject is taught which behavior is correct, and remember, we talked about shaping. Shaping uses operant conditioning principles to train a subject by rewarding proper behavior and discouraging improper behavior. It’s imperative to do that early to establish the proper behaviors.
  2. The schedule is best matched for the motivational state of the subject. This answer choice is out of scope and is not going to affect the specific question being asked here.
  3. This leads to the greatest accumulation of reinforcements, hence the greatest pleasure. While we do want the greatest accumulation of reinforcements, this does not correlate with the greatest pleasure. Think about how bad behaviors are discouraged. There is not the same pleasure we might see in positive reinforcement.
  4. It gives subjects little time for incorrect responses. While we would ideally have few incorrect responses, we do expect some incorrect responses. That’s how operant conditioning and learning happens. We’re going to stick with answer choice A as our correct answer. 

14) It’s important to keep the key points straight here. We have two children with different parents. These children are adopted very young, so extremely early in their development. We want to look at heredity as a determiner of intelligence, so we’d think about the transmission of genetic information more than the subsequent environment.

  1. are more similar to each other than to their own biological siblings. If this were true, this would be an argument against the idea that heredity is an important determiner of intelligence. This instead would support the idea that environment post-birth is a more important determiner of intelligence.
  2. are less similar to each other than to their own biological siblings. This is consistent with our breakdown and supporting the idea that heredity is an important determiner of intelligence. Despite having biological siblings, the boys’ IQs are more similar to those of their adopted siblings. That means environment is more important than their genes.
  3. are more similar to their adoptive parents than to their biological parents. This is similar to answer choice A. This would support the idea that environment post-birth is a more important determiner of intelligence. We can stick with answer choice B as our best option.
  4. are less similar to each other than to their adoptive siblings. This is not answering the specific question being asked. We’re comparing adopted siblings to different adopted siblings. This does not consider biological siblings or the idea that heredity is an important determiner of intelligence. Answer choice B is going to be our best answer.

15) To answer this question, we can visualize what’s happening. We have an animal trainer dealing with a rabbit, but this is no ordinary rabbit! This rabbit is going to be a television star in a commercial. While it might be easy to train this rabbit in a calm environment away from distractions, we have to know what the trainer will have to avoid when training this rabbit for a television commercial. What does that entail? Actors, cameras, and a lot of commotion. We have to make sure the rabbit remembers its training and doesn’t follow its animal instincts and accidentally go “off-script.”

  1. Operant extinction. Extinction, in operant conditioning, refers to when a reinforced behavior is extinguished entirely. This occurs at some point after reinforcement stops; the speed at which this happens depends on the reinforcement schedule, which is discussed in more detail in another section. Considering there is still training happening (according to the question stem), this is not a viable option. 
  2. Instinctual drift. Instinctive drift occurs when organisms tend to revert to unconscious and automatic behavior that could interfere with learned behaviors from operant conditioning. This is a valid phenomenon the trainer will avoid. It’s tough for a trainer to train a rabbit to go against its instincts, so the trainer will likely try and avoid any instances where instincts may kick in. 
  3. Stimulus generalization. This involves demonstrating the conditioned response to stimuli that are similar to the conditioned stimulus. Presumably the trainer will be explicit in giving any instructions and this will not be an issue. Answer choice B remains the best option. 
  4. Partial reinforcement. In partial reinforcement, behavior is reinforced in certain intervals. This is in contrast to continuous reinforcement which involves reinforcing behavior every time. This answer choice doesn’t answer the specific question being asked. Partial reinforcement involves learning behaviors more slowly, but this is not going to affect the training of our TV star! We can stick with answer choice B as our best answer.

16) We’ve all been in this situation. We forget something in our car or our house, we walk away, and then realize the mistake we made. That’s when a big decision must be made. Do we go back and get it, or do we keep walking? The person in our question stem made the easy choice. He felt he’d come too far, so he kept going into the store. Why did he only remember items from the two ends of the list? This is a classic vocabulary question. By thinking about a string of events or even words, it is possible to use a previous memory to cue the next item in the series. This can sometimes lead to serial-position effect which is the tendency of a person to recall the first and last items in a series best, and the middle items worst.

  1. the serial-position effect. This matches our breakdown and the definition of the serial-position effect perfectly. This is likely going to be our correct answer, but we can keep comparing.
  2. hindsight bias is the belief that the event just experienced was predictable, even though it really wasn’t. This is out of scope here. Forgetting the items from the middle of the list is not because of hindsight bias. 
  3. chunking. This is another definition. Chunking is the process of organizing parts of objects into meaningful wholes. The whole is then remembered as a unit instead of individual parts. Examples of chunking include remembering phone numbers (a series of individual numbers separated by dashes) or words (a series of individual letters).
  4. decay. This has to do with the idea that memories that aren’t recalled constantly, they will decay and eventually be forgotten. While this could be valid if the entire list was forgotten, that wasn’t the case. We can stick with answer choice A as our correct answer because of our breakdown of the question.

17) This question is asking us which of the 4 answer choices will not be diminished as a person ages. Three of the answers will decline with age, while one will either remain unchanged or improve. Things like implicit memories and recognition will not diminish. Semantic memory, emotional reasoning, and crystallized intelligence can improve over time. Alternatively, things like episodic memory, recall, and prospective memory tend to decrease over time. We want something from the former list.

  1. capacity for acquiring new declarative information. As a person ages, it does become more difficult to acquire new declarative information. While older people can recall older information, the key word here is “new”.
  2. capacity for controlling his or her memory processes. We have to be careful with the verbiage here because we’re not focused on recalling memorize themselves. We’re considering controlling his or her memory processes. Controlling implies actually manipulating these memories, which is not as easily done as we age.  
  3. ability to cope with Alzheimer’s Disease. As a person ages, they are more vulnerable to disorders like Alzheimer’s disease. This ability decreases with age. 
  4. ability to retrieve general information. This is something we actually mentioned as we went through answer choice A. Older people can retrieve general information, even though acquiring new declarative information is difficult. This is going to be the best answer choice as answer choices A-C all listed things that will diminish with age.

Section Bank: Psychological, Social, and Biological Foundations of Behavior: Passage 3

18) To answer this question, we’re focused on how memories typically work, versus how this reminiscence bump occurs with music. 

The author says older adults have better memory for events that occurred when they were between 10- and 30-years old, than during any other time period. Why is that atypical? It’s typically easier to remember recent events than those further in the past. Those further memories tend to decay and fade away over time. We expect the adults to have better memory for the most recent events that occurred. 

  1. enhanced memory performance in an otherwise increasing retention function. This answer choice implies that adults have increasing retention function for the memories from between 10 and 30-years old. That’s not normally the case. We just mentioned it’s easier to remember more recent events, so this answer choice contradicts our breakdown. 
  2. enhanced memory performance in an otherwise decreasing retention function. This matches our breakdown, and what we said was the reason behind our study. Normally, memories decay and fade over time. That’s decreasing retention function. In this case, there’s actually enhanced memory performance. We like this answer choice better than option A. 
  3. reduced memory performance in an otherwise increasing retention function. This answer choice contradicts our breakdown and the passage right away. The reminiscence bump represents increased memory performance. We mentioned older adults have better memory for events that occurred when they were between 10- and 30-years old. We’re also looking for an answer that mentions decreasing retention function. 
  4. reduced memory performance in an otherwise decreasing retention function. Reasoning here is the same as answer choice C. There older adults have better memory for events that occurred when they were between 10- and 30-years old which is atypical. We can eliminate this answer choice for contradicting the passage and our breakdown. We’re left with our correct answer, answer choice B. 

19) To answer this question, we’ll rely on vocabulary. Vocab is a huge part of this section. Often, you’ll see pseudo-discrete questions or pseudo-standalones. Even though the test-maker might ask you about something related to the passage, you don’t technically need a lot of information from the passage to answer. 

We’re dealing with personal, long-term memories. First of all, we should distinguish between implicit and explicit memories. Explicit memories we can consciously remember or recall. These include episodic memories of events/experiences, and semantic memories of common knowledge or concepts. Implicit memories are less conscious. They include procedural memories like how to ride a bike and emotional conditioning. We’re looking for an answer choice that mentions explicit, or declarative memories.

  1. Episodic. Episodic memories we just mentioned are a kind of explicit memory. These include specific events and episodes. That’s exactly what we’re looking for in the question stem. There’s a memory around a specific experience or episode. We’ll keep this answer choice as a good option.
  2. Semantic. Semantic memories deal with ideas and concepts that aren’t from personal experience. That’s a big distinction between A and B here. This contradicts what we were asked in the question stem, so we can eliminate answer choice B.
  3. Implicit. Implicit memories, which we also talked about, are not consciously remembered. These are things like procedural memories and emotional conditioning. We said in our breakdown, this isn’t what we’re looking for, so we can eliminate this answer choice.
  4. Sensory. Sensory memory is a brief storage of information and lasts only a few seconds. Not what we’re looking for in our question stem. We can eliminate this answer choice as it’s unreasonable. We’re left with our correct answer, answer choice A: episodic.  

20) To answer this question, we want to summarize the correlations from the study, but within the context of the participants. 

We have an excerpt from the passage here, it says “Number of personal memories was positively correlated with the percentage of songs recognized and with whether participants liked the songs.” This is the main correlation studied in the passage, and it’s consistent with pretty much everything else the author says up to this point.

  1. to like the music that evoked personal memories more than music that did not evoke personal memories. This is 100% consistent with the passage and our breakdown. Personal memories positive correlated with songs recognized and whether the participants liked the songs. We can hold on this answer choice for now. 
  2. to prefer music chosen by themselves and their peers over music chosen by their parents. This answer choice is interesting, because participants had positive memories associated with both of these. However, there’s no mention in the passage that the participants preferred one brand of music over the other. This is making a connection that isn’t actually there, so we can eliminate answer choice B.
  3. to prefer music associated with positive emotions over music associated with negative emotions. Even though the researchers tracked the degree to which the participants felt different emotions, there’s no mention of liking music associated with one emotion more than music associated with a different emotion. Another connection that the researchers and the author didn’t make in the passage.
  4. to like the music that their parents listened to as much as music they heard more recently. This is similar to answer choice B. It’s making a connection that isn’t actually there. We don’t know if the participants prefer the music their parents liked versus the music they heard more recently. We’re left with our correct answer, answer choice A. 

21) This is a very open-ended question, meaning we’ll likely go through our answer choices and consider each one within the context of the passage. 

  1. a subset of counterculture. A counterculture is a group whose members adopt cultural patterns in opposition to the larger culture. There’s nothing in the passage implying this is true. Music in the context of this passage has nothing to do with counterculture.
  2. an agent of socialization. Agent of socialization is something that can impress social norms upon an individual. That can include people such as family, friends, colleagues, and neighbors. It can include social institutions like church and school, or consumption of mass media. By that definition, popular culture is an agent of socialization. We like this answer choice for now. 
  3. a subset of material culture. Material culture refers to the relationship between artifacts and social relations. So essentially, it deals with physical objects. Not a viable answer here. 
  4. an agent of social reproduction. This refers to the emphasis on the structures and activities that transmit social inequality from one generation to the next. In this passage, the only real thing being transmitted is musical taste, not social inequality. This answer choice is also not viable here. We’re left with our correct answer, answer choice B: an agent of socialization. 

22) To answer this question, we can go back to our results and see why we observe two reminiscence bumps. 

We know from the passage, adults recognize the music that they encountered during their late adolescence and early adulthood more easily than music they encountered later. That’s the usual reminiscence effect. But we also have a second reminiscence effect base on when their parents were between 20 and 25 years of age. We’re told that over time, participants’ memories shifted from listening to music chosen by their parents to music chosen by themselves and peers. We want an answer choice that explains this shift.

  1. social status. This refers to one’s standing in the community and position in the social hierarchy. This may or may not affect musical preferences, but we’re focused on what was discussed in the passage. No mention of social status in the passage, let’s keep comparing.
  2. front stage self. This is the behavior a player performs in front of an audience, or society. Meaning when the person knows they’re being watched. Within the context of the passage, the front stage self and the backstage self are not going to shift musical preferences. 
  3. social network. We touched on this briefly. When you’re younger, your social network revolves around your family and who you live with. As you go to school and become more independent, your social network expands. This explains what we said in the passage: that participants’ memories shifted from listening to music chosen by their parents to music chosen by themselves and peers. We like this answer choice. We can eliminate answer choices A and B. Neither explains the shift in musical preferences based on the passage. 
  4. back stage self. This is the opposite of answer choice B: when players are together, but no audience is present. In other words, a more private setting where you’re not being watched. Again, within the context of the passage, the front stage self and the backstage self are not going to shift musical preferences.

23) This answer is going to come from knowing the definition of reminiscence bump in the passage and relating it to behavioral vocab words. Reminiscence bump in the passage is related to the fact that older adults have better memory for events that occurred when the subject was between 10 and 30-years old. There’s also a second reminiscence bump for music when parents were between ages 20 and 25. Despite the fact adults are older, they still remember music from when they were in that 10 to 30-year old range. Even more easily than the music they encounter later. 

  1. culture lag. This is a concept related to the fact that culture takes time to catch up with technological innovations, resulting in social problems. This is out of scope and doesn’t describe the reminiscence bump.
  2. cultural assimilation. This is the process by which an individual or group becomes part of a new culture. Again, not something that describes the reminiscence bump. We’re still holding on to A and B, although both are out of scope at first glance.
  3. culture shock. This is the feeling of disorientation that occurs due to an encounter with an unfamiliar culture. This might happen when moving to a new country or area. Not relevant to the reminiscence bump we read about in the passage. 
  4. cultural transmission. Transmission is the passage of culture from one generation to another. This is exactly what we saw with music. There was a second reminiscence bump for music that was popular even before the participants were born. This is unusual, but the researchers are able to explain this by mentioning this music was recorded when the parents were between ages 20 and 25. That’s consistent with reminiscence bump. We’re left with our correct answer, answer choice D: cultural transmission. 

Section Bank: Psychological, Social, and Biological Foundations of Behavior: Passage 4

24) We can go back to the passage and see where the author talks about this specific stimulus. Just like so many other psych/soc questions, this ultimately comes down to knowing your vocab.

It says in many cases, people’s perceptions of objects, scenes, and events in the world differ from the stimulus that is registered by the sensory receptors. We’re focused on the stimulus mentioned in the last bit of the sentence. Let’s break down the whole process. There’s an object, scene, or event in the external world. But that’s not what’s being registered by the sensory receptors. Instead, that object or event will reflect light or energy. It’s that light or energy that excites our sensory receptors. We’re focused specifically on that light that actually excites our receptors, and leads to perception. 

  1. incentive stimulus. This is trying to get into incentive theory. That just says behavior is extrinsically motivated. People perform activities if they have a reward or incentive. That’s out of scope in this situation. 
  2. sensory stimulus. This is referring more to the object or event that’s received by the senses, and eventually leads to a response. Remember, we’re looking for the stimulus actually registered by the sensory receptors. This is better than answer choice A, but still not addressing what we’re looking for specifically.
  3. distal stimulus. In light processing, the distal Stimulus is any external physical object or event that reflects light, and the light itself is the proximal stimulus. The proximal stimulus is what excites the receptors on our eyes, leading to visual perception. 
  4. proximal stimulus. This is what we just brought up in our breakdown. We have an object, scene, or event in the external world, but that’s not what’s being registered by our sensory receptors. Instead, the object reflects light, and that light excites our sensory receptors. It’s that light that’s the proximal stimulus. Answer choices B and C contradict this definition. We can stick to our best answer, answer choice D: Proximal stimulus.

25) We’ll pull up column D and we’ll use our vocab to explain why we see the items grouped in columns.

We have Figure 1 from the passage here. We’re focused on Panel D. And the author says here, people tend to see the items arranged in columns of squares and circles, rather than rows of alternating shapes. Said differently, all we’re doing here is finding a vocab word that matches this grouping. We can look at Gestalt laws of grouping. We’re naturally perceiving organized columns of circle and square columns. This specific example is the Law of similarity. Similar elements are perceptually grouped. We also see the classic example of the figure-ground law in C. But for our sake, we’re going to stick with the Law of similarity.

  1. Similarity. This matches our answer choice perfectly. But our job’s not done yet; we still want to compare to our other answer choices and make sure we’re picking the best answer here. 
  2. Proximity. Proximity deals with objects being close to each other and being grouped together. Everything is the same distance in panel D, so not a legitimate explanation, we can eliminate this answer choice.
  3. Closure. Closure means missing information is filled in to complete a figure. That’s not applicable for panel D. We can also eliminate this answer choice for contradicting our definition.
  4. Continuity. Law of Continuity explains that lines are seen as following the smoothest path. Not applicable in this case. We can eliminate this answer choice as well. We’re left with our best answer, answer choice A: Similarity.

26) We want to actually group the principles of perceptual organization in Paragraph 1 into a single psychological theory. I’m sure a lot of you picked up on the fact that we did this in our last question; this is a straight vocabulary question. Gestalt laws of grouping explain how humans naturally perceive stimuli as organized patterns and objects. That’s exactly what we’re doing in the passage, and for our last question.

  1. Behaviorist. Behaviorist perspective regards individuals’ actions as being responses to external stimuli and driven by outcomes. This is out of scope.
  2. Gestalt This matches our breakdown exactly and we already gave a definition. We can eliminate answer choice A now. Answer choice B is now our best answer.
  3. Humanistic. The humanistic perspective Argues that an individual’s subjective free will is the most important determinant of behavior. Humans strive to be their ideal self. This is out of scope.
  4. Cognitive. Cognitive theory functions to explain human behavior by focusing on your mind and your thoughts. These thought processes determine emotions, behavior, and personality. Again, not as relevant to organizing patterns and shapes. We can also eliminate this answer choice for being out of scope. We’re left with our correct answer, answer choice B: Gestalt.

27) From the 4 experiences given in the answers, we want to find the one that’s not related to the principles of perceptual organization described in the passage. Again, another vocab-heavy question. We don’t know what any of the experiences are just yet, and predicting every possible experience isn’t feasible. We’re going to jump into our question, but with the mindset of relating the experience to the principles of perceptual organization that were described in the passage.

  1. Seeing a portion of food served on a large plate as being smaller than the same portion of food served on a small plate. This is an optical illusion of relative size perception. This was Panel A in Figure 1, and the author described this in the passage. This contradicts what we’re looking for.
  2. Seeing birds flying in the same direction as being part of an integrated flock. This goes back to Gestalt which we covered extensively in the passage and questions. Humans perceive visual elements that move in the same speed or direction as parts of a single stimulus. The flock of birds is the classic example of this.
  3. Seeing cars lined up in a parking lot as being in long rows, rather than in pairs facing each other with a gap between each pair. Back to Gestalt and something we just talked about in question 25. Similar elements are perceptually grouped. This also goes against what we’re looking for.
  4. Seeing a word with a missing letter and being able to identify the word, based on the sentence in which it is contained. This is describing context effect. We can use prior experiences and knowledge to analyze familiar scenes and objects. In this case, you’re using prior knowledge to fill in the most probable word. This isn’t something we addressed in the passage, so that makes this a great option. We eliminate answer choice A-C, those all relate to what we covered in the passage. We’re left with our correct answer, answer choice D. 

28) To answer this question, we want a methodology that can assess our perception of stimuli like the ones in Panel A. That means revisit Panel A, then we’ll jump into different methodologies using our general knowledge. We want to show how perceptual illusions, like the one in panel A, impact our judgments of the nature of stimuli.

Which psychological theory best explains the role of community values in the preliminary studies of tobacco cessation?

We have Panel A up top. We also have the part of the passage that talks about panel A. It says the introduction of other stimuli around the perimeter of two central dots, as shown in Panel A, can lead to the misperception of the central dot in the left half of the figure as being smaller than the central dot in the right half of the figure. We’re interested in how we can assess the impact on our judgment here. There’s obviously an illusion going on. Despite the central dots being the same size, we perceive the right central dot as bigger. We’re looking for the methodology that can assess the impact on our judgment in this situation. This is all going to be vocab-based.

  1. Partial report technique. This involves recalling a partial amount of the total information presented to a subject. More to do with memory. That wouldn’t be as applicable for this perceptual illusion.
  2. Word association testing. This requires the subject to respond to a series of words with the first word that comes to mind. In psychology this can be used to explore individual differences like thoughts and traits, but again, not as relevant for our perceptual illusion. 
  3. Psychophysical discrimination testing. This involves asking participants to differentiate between stimuli with just noticeable differences. For this passage, that can mean making the circles around the central dot slightly different sizes. Eventually we notice the perceived difference. I’m liking this answer choice. We can eliminate answer choices A and B because we said both were out of scope.
  4. Operational span testing. This is used to test the capacity of working memory, so similar to answer choice A that focuses on memory. Another answer choice that is out of scope, we can eliminate answer choice D. We’re left with our correct answer, answer choice C: Psychophysical discrimination testing. 

29) To answer this question, we have to determine which part of panel A can impact our judgment and decision-making, but are not an actually a part of those processes. We’re picking from 4 possible perceptual experiences that are illustrated in panel A; we’re ultimately relying on our vocab to decide which answer choice is most relevant to Panel A.

Which psychological theory best explains the role of community values in the preliminary studies of tobacco cessation?

We have Panel A up top. We also have the part of the passage that talks about Panel A. It says the introduction of other stimuli around the perimeter of two central dots, as shown in Panel A, can lead to the misperception of the central dot in the left half of the figure as being smaller than the central dot in the right half of the figure. Adding these circles around the central dot impacts our perception. 

  1. recall cues. This has to do with recall memory. Cues are given to remember information, material, or memories. Not relevant to our question here, or the example with the dots.
  2. context effects. Context effects have to do with relationships and using prior experiences and knowledge to interpret a stimulus. They involve using environmental cues to help examine the stimuli. These external cues can influence how we view the stimuli. That’s exactly what’s happening in Panel A. Central dots are the same size, but the dots surrounding them make up the context in which the central dots are presented. That context influences our judgment. We like this answer choice, we can now eliminate answer choice A, it was out of scope.
  3. feature detectors. Neurons respond to aspects of visual stimuli. Cells provide information about the basic features of objects. This information is integrated to produce a perception of the object as a whole. Feature detection is a type of serial processing where increasingly complex aspects of the stimulus are processed in sequence. Also out of scope.
  4. practice effects. The old saying goes “practice makes perfect,” implying that practice will make you better at a task. That’s true in these psychology tests also. Participants’ performance on a task can improve from repeating the activity…instead of from any study manipulation imposed by the researcher. Not as relevant here, we can eliminate answer choice D for being out of scope. We’re left with our correct answer, answer choice B: Context effects.

Section Bank: Psychological, Social, and Biological Foundations of Behavior: Questions 30-34

30) This is something a lot of people can relate to unfortunately. In this case, however, we’re just identifying the vocabulary word describing the phenomenon. Let’s define our 4 terms and consider each one.

  1. Repression. This describes actually repressing or blocking out a memory that is too painful or traumatic. That’s not what is happening in the question stem. In fact, details are being remembered more vividly than usual. Let’s look for a better answer. 
  2. Flashbulb memory. This is going to be a good option. A flashbulb memory is a highly detailed, exceptionally vivid ‘snapshot’ of the moment and circumstances in which a piece of surprising and consequential news was learned. This matches what we’re looking for, so we can eliminate answer choice A. 
  3. Recency effect. This is an effect in which the items that were presented last are also recalled well as they are still being held in the working short-term memory. This term is out of scope here. We’re not focused on recency effect.
  4. Spacing effect. Spacing effect allows a person to remember something they have studied many times spaced over a longer period of time rather than all at once. Again, not as relevant as answer choice B. We can stick with option B as our correct answer.

31) This is a standalone question that relies on using external knowledge. We get a little physiology here to mix with our psychology! We have to be careful with the verbiage here. The test-maker mentions a theory that hunger drive is based on a person’s interpretation of stomach contractions and satiety is based on stomach distension. Do we want an answer choice consistent with this theory? No! We want an observation that disconfirms this theory.

  1. Stomach contractions do not correlate with the experience of hunger. This is relevant to the information in the question stem. This correlation provides some evidence against the theory, but doesn’t fully disconfirm it.
  2. Stomach distension does not correlate with satiety. Reasoning here is going to be identical to answer choice A. Because both A and B are equally correct, we will likely end up eliminating both. Every MCAT question can only have one correct answer!
  3. Rats without stomachs cannot learn mazes when rewarded with food. This relationship has more to do with learning, not as much with hunger or satiety. This answer choice is irrelevant, so we can eliminate it. 
  4. People whose stomachs have been removed still experience hunger. This is extreme, but certainly a way we can disconfirm the theory in the question stem! If there is no stomach, but the person still experiences hunger, then we can say that hunger is not coming from the stomach. Answer choice D is going to be our correct answer.

32) Like many other behavioral questions, specifically standalone questions, this is going to come down to knowing your vocabulary. There’s no related passage, so we’ll go through the definition of the four answer choices and decide which one is the one described in the question stem. 

  1. Autobiographical memory. An autobiographical memory is recalled from one’s own life. While this may or may not apply to the specific memory being reported, the definition in the question stem does not describe an autobiographical memory. Let’s try and find a superior answer. 
  2. False memory. This occurs when people remember events differently from the way they happened or, in the most dramatic case, remember events that never happened at all. False memories can be very vivid and held with high confidence, and it can be difficult to convince someone that the memory in question is wrong. They have not forgotten the event but have constructed events that either never happened or happened in a different order. This sounds like a good option for now. 
  3. Amnesia. This is a deficit in memory, can be caused by brain damage. Anterograde amnesia is the inability to store new memories; retrograde amnesia is the inability to retrieve old memories. 
  4. Recovered memory. This involves recalling a memory that was previously repressed or not remembered. It does not necessarily have to be inaccurate or expressed with extreme confidence. Answer choice B is going to be our best option.

33) Kohlberg defined three levels of moral development: preconventional, conventional, and postconventional. Each level has two distinct stages.

Level 1: Preconventional: Throughout the preconventional level, a child’s sense of morality is externally controlled. Children accept and believe the rules of authority figures, such as parents and teachers.

Level 2: Conventional: Throughout the conventional level, a child’s sense of morality is tied to personal and societal relationships. Children continue to accept the rules of authority figures, but this is now due to their belief that this is necessary to ensure positive relationships and societal order.

Level 3: Postconventional: Throughout the post-conventional level, a person’s sense of morality is defined in terms of more abstract principles and values. People now believe that some laws are unjust and should be changed or eliminated. This level is marked by a growing realization that individuals are separate entities from society and that individuals may disobey rules inconsistent with their principles.

Which psychological theory best explains the role of community values in the preliminary studies of tobacco cessation?

  1. To gain acceptance. This is in Level 2: the conventional stage of moral development.
  2. To follow rules. This is also going to be in Level 2. We see obeying authority and conforming to social order.
  3. To promote social welfare. Unlike answer choices A and B, this is in Level 3: the post-conventional stage. The moral principles involved make this a promising answer choice.
  4. To avoid disapproval. This is in Level 1: the person is avoiding punishment. We can eliminate this answer choice and stick with our correct answer, answer choice C: to promote social welfare.

34) To answer this question, we can define operant conditioning and then jump into our four options. We’re focused on motivation. We can break down what we know about operant conditioning: it’s a type of associative learning process through which the strength of a behavior is modified by reinforcement or punishment. When we deal with operant conditioning, we think of shaping which involves a calculated reinforcement of a “target behavior”: it uses operant conditioning principles to train a subject by rewarding proper behavior and discouraging improper behavior. The method requires that the subject perform behaviors that at first merely resemble the target behavior; through reinforcement, these behaviors are gradually changed or “shaped” to encourage the target behavior itself.

Which psychological theory best explains the role of community values in the preliminary studies of tobacco cessation?

Note, in the context of operant conditioning, whether you are reinforcing or punishing a behavior, “positive” always means you are adding a stimulus (not necessarily a good one), and “negative” always means you are removing a stimulus (not necessarily a bad one). 

  1. observing the subject’s behavior over a long period of time. This could be helpful in some circumstances, but it has little to do with the reinforcement or punishment on which we’ve been focusing. Let’s try and find a more relevant answer.
  2. using a type of reinforcement that the experimenter knows the subject usually likes. Using a type of reinforcement the subject likes is more for conditioning and less to test motivation. Motivation is typically done through deprivation. 
  3. depriving the subject of some desirable stimulus item for a period of time. You might have experienced this first-hand. If you like something a lot and happen to lose it, you will want to get it back. If they take away one of these stimuli, the researchers can see what subjects desire/are motivated by the most. We like this answer choice for the time being. Whenever we see deprivation tied to operant conditioning or shaping, we should at least consider relationship to motivation. 
  4. using a novel stimulus that the subject is sure to like. One thing that stands out here is a “novel stimulus that the subject is sure to like” is a contradiction. There is no real way to know a subject will like a novel stimulus. This also ties into answer choice B. This helps more in conditioning, but not so much in researching motivation. We can stick with answer choice C as the best answer.

Section Bank: Psychological, Social, and Biological Foundations of Behavior: Passage 5

35) The researchers in the studies will support a sociological perspective on health or illness, but we want to be careful that we use what’s told to us in the passage based on the verbiage in the question stem. However, we’ll likely need to use our vocab as well. We want to define the 4 answer choices, and see which one matches our criteria. We want to see if the researchers in the studies will support these sociological perspectives.

Researches in Study 1 assessed whether Stroke Belt residence was linked to excess risk of stroke later on. Researchers in Study 2 did something similar, but they focused on specific age ranges and length of time. That allowed them to pinpoint specific ages that correspond to increased risk of stroke. We want a sociological perspective on health or illness that the researchers in each study would support. 

  1. The medicalization of illness. This refers to conditions and problems being defined as medical conditions. A behavior or condition that was previously not considered an illness or a medical condition…that is now defined and treated as a medical condition. Neither set of researchers touched on this or conducted their study in a way that would support this perspective.
  2. The life course approach to health. Life-course theory analyzes people’s lives in the context of social, structural, psychological, and biological perspectives. The life-course approach to health is therefore going to look at environmental factors, life experiences, and social environment and how it affects their health. That means looking into childhood, adolescence, adulthood, and all ages to tie it to health. Both researchers assessed whether Stroke Belt residence and exposure was linked to excess risk of stroke later on. That means. Both sets of researchers would support this perspective. This answer choice is superior to answer choice A.
  3. The socioeconomic gradient in health. This has to do with income inequalities corresponding to health differences. Both sets of studies actually accounted for any socioeconomic differences. This wouldn’t affect what they found in their studies or observations. We can also eliminate this answer choice, it’s out of scope.
  4. The social construction of illness. Social constructionism focuses on how knowledge and experiences aren’t real. Instead, they exist because individuals and society give them meaning. Not something the researchers imply or something the author talks about during the passage. We can eliminate answer choice D, it’s also out of scope. We’re left with our correct answer, answer choice B: The life course approach to health.

36) We can go back to the passage to get some context about the responses to survey questions, but this answer is ultimately going to come from knowing content.

Which psychological theory best explains the role of community values in the preliminary studies of tobacco cessation?

We have excerpts from both studies in the passage here. We’re focused on validity of responses to the surveys. For Study 1 it says Data were from a longitudinal survey of adults over 50 years old. For Study 2, it says Data were from a longitudinal survey that tracked stroke incidence among adults over 45 years old. Both longitudinal surveys. A longitudinal study means researchers follow this specific population at multiple time points. That’s in contrast to a cross-sectional study that looks at one point in time. We have 50 and 45-year olds surveyed in the study. They’re asked to recall information about their residential history, meaning specific events and experiences. We want an answer choice that deals with explicit memories, and specifically episodic memories.

  1. Proactive interference. Proactive interference is when memories from someone’s past influence new memories. Past memories are not hindering subsequent learning in this case. This answer choice is out of scope. 
  2. Source amnesia. This is the inability to remember where, when, or how previously learned information has been acquired, while retaining the factual knowledge. You can’t remember the actual source. Not something that affect the validity of the responses. This is also out of scope, 
  3. Decay in episodic memory. This actually matches our breakdown exactly. Episodic memories are long-term memories that are consciously remembered. These include explicit and implicit memories. Within explicit memories are episodic memories, or memories of specific events, experiences, or episodes. This matches our breakdown. The participants are asked about where they lived in the past. They’re trying to use episodic memory. A decay in episodic memory would affect the validity of their responses. We like this answer choice, we can eliminate answer choices A and B, those are out of scope. 
  4. Decay in semantic memory. Semantic memory has to do with more general knowledge. Knowledge about the world, and not specific events, experiences, or episodes. This contradicts what we said in our breakdown of the question. We can eliminate answer choice D, we’re left with our correct answer, answer choice C: Decay in episodic memory. 

37) To answer this question, we have to decide which of the findings from the follow-up studies would support the hypothesis at the end of Study 1. 

We have an excerpt from Study 1-it’s the last paragraph that talks about the hypothesis and follow-up studies. Hypothesis is: “genetic predispositions to stroke may be more likely among people living in the SB.” The follow-up study considers twins born in SB states, but adopted by two different families. If there’s a genetic predisposition to stroke, both twins can grow up in different environments but ultimately still have elevated risk of stroke as adults. 

  1. Monozygotic twins who were raised in the same state outside of the SB both have decreased risk of stroke in adulthood. This answer choice is the opposite of what we want. In this case it’s providing evidence against a genetic predisposition to having a stroke. 
  2. Dizygotic twins who were raised in the same state inside of the SB both have excess risk of stroke in adulthood. This answer choice at lease talks about excess risk of stroke in adulthood, but doesn’t quite provide evidence for the hypothesis. Both twins were raised in the same state, so they have a similar environment that normally causes an excess risk of stroke in adulthood. Also, we have dizygotic twins. They only share 50% of the same genes, not 100% like monozygotic. This is superior to answer choice A, which we can now get rid of. That contradicted our breakdown.
  3. Monozygotic twins who were raised in different states, one twin inside the SB and one twin outside, both have excess risk of stroke in adulthood. This answer choice is consistent with our breakdown. We have twins raised in different states, but both end up with excess risk of stroke in adulthood. This is despite being raised in different states, and only one twin inside the Stroke Belt. Both have the same genes, but different environments. The reason both have excess risk of stroke in adulthood can be that genetic predisposition. We like this answer choice, we can eliminate answer choice B.
  4. Dizygotic twins who were raised in different states, one twin inside the SB and one twin outside, both have excess risk of stroke in adulthood. Here we have dizygotic twins. They only share 50% of the same genes, not 100% like monozygotic. Answer choice C is a better choice because we know both twins will share 100% of the genes. We can eliminate answer choice D. We’re left with our correct answer, answer choice C.

38) Researchers adjusted for demographic characteristics like age, race, gender, ethnicity, and religion among other characteristics. There was adjustment for socioeconomic status which is a person’s place in a class structure: income, occupation, wealth, and education level. And also, cardiovascular risk factors.

  1. The adjustment determined whether the results from the sample were generalizable. This has more to do with applying the results of the study elsewhere, or in different studies. Controlling for factors doesn’t determine whether the results were generalizable or not. This is out of scope. 
  2. The researchers controlled for factors such as social capital and cultural capital. Social capital is the networks of relationships among people who live and work in a particular society. Cultural capital is the accumulation of knowledge, behaviors, and skills that a person can tap into. These demonstrate one’s cultural competence and social status. We can argue the statistical adjustment did control for cultural capital, but not as much social capital. Still a better option than answer choice A. 
  3. The adjustment determined whether the measures of stroke risk were reliable. The statistical adjustment doesn’t actually change the reliability of stroke risk. The data and measurements remain the same, despite adjustments. We can eliminate this answer choice.
  4. The researchers controlled for factors such as race/ethnicity and education. All the factors listed in this answer choice actually match the factors we mentioned in our breakdown of the question. The researchers controlled for these factors in both studies. This is a better answer than answer choice B. We’re left with our correct answer, answer choice D.

39) What was the conclusion for Study 1? That’s in the middle paragraph. It says “The researchers concluded these findings were consistent with other studies that suggest a link between early development and physiological or behavioral changes that increase the risk of stroke decades later.” Stroke Belt residence in childhood was, in fact, linked to excess risk of stroke in adults. We want to find an answer choice that’s consistent with this childhood period being related to the increased risk of stroke, even decades later.

  1. Insecure attachment. Insecure attachment is likely formed in early childhood. It’s a level of fear or uncertainty in relationships. This has more to do with social competence, not as much with the increased risk of stroke in adults because of residence in the stroke belt.
  2. Sensitive period. These are periods in which individuals and specifically children, can acquire certain skills or characteristics. That sounds like exactly what we’re told in Study 1. Early development and physiological changes can increase the risk of stroke decades later. The conclusion revolves around this early, sensitive period. We can now eliminate answer choice A for being out of scope. 
  3. Assimilation. This is the process of making new information fit in with your existing understanding of the world. This is out of scope. The increased stroke risk is not due to assimilation. 
  4. Modeling. Modeling has to do with learning through imitation. A person will observe behavior of another person and then imitate. This isn’t the developmental concept on display in Study 1. We’re left with our correct answer, answer choice B: Sensitive period. 

40) This is similar to our last question. For this question, the conclusion in Study 2 is going to be consistent with one of the 4 public health strategies in Stroke Belt states. Let’s take a look at an excerpt from the passage.

It says The researchers concluded that the significance of adolescence may be related to establishing social norms that will affect stroke risk in adulthood. We know specific age ranges and lengths of time are associated with increased stroke risk. In particular the strongest association with stroke incidence was found for Stroke Belt exposure during adolescence. This is when individuals establish social norms that affect stroke risk in adulthood.

  1. Targeting the formation of health behaviors in social networks. This answer choice is consistent with what we read in the passage. We mentioned in adolescence, individuals will establish social norms, and that can include the health behaviors they see and experience in their social networks. We like this answer choice for now.
  2. Eliminating prejudice and ethnocentrism in health care delivery. This is something that the researchers actually adjusted for. They adjusted for demographic factors, socioeconomic status, and cardiovascular risks. The conclusion revolved around the fact that individuals will establish social norms in adolescence. This can include health behaviors that later contribute to stroke risk. 
  3. Reducing role conflict and supporting psychosocial development. This sounds like a very positive strategy, but it’s out of scope. This is not something that would address the social norms being established and affecting stroke risk in adulthood.
  4. Increasing social and cultural capital in low income communities. This is similar to answer choice B. Socioeconomic status was adjusted for. Instead, the conclusion revolved around the fact that individuals will establish social norms in adolescence. This can include health behaviors that later contribute to stroke risk. We can eliminate this answer choice. We’re left with our correct answer, answer choice A: Targeting the formation of health behaviors in social networks. 

Section Bank: Psychological, Social, and Biological Foundations of Behavior: Passage 6

41) In the passage, we were introduced to the studies, now we want to relate it to our behavioral vocabulary. Just like the author mentions, studies suggest that tobacco cessation is more successful when motivated by culturally consistent values that support the effort to quit smoking. We want to look at different theories that explain how motivation affects human behavior. We want a theory that mentions being motivated by the norms of the culture, and acceptance.

  1. Drive theory. Drive reduction theory involves being motivated to satisfy physiological needs, and maintain homeostasis. Think thirst or hunger. The need to stop smoking is externally motivated, and not something the individual feels is necessary to maintain homeostasis. 
  2. Humanistic theory. This has to do with the idea that we’re motivated to fulfill lower, base requirements first and foremost. As these lower level requirements are fulfilled, we can progress toward self-actualization. In this case, the smoking is an extrinsic motivator that is instead tied to cultural values.
  3. Incentive theory. This argues that behavior is primarily extrinsically motivated. There’s a desire, need, or drive to stop smoking in this case. That will allow them to follow the norms of the culture and fit in and be accepted. The individuals are motivated by culturally consistent values, or external motivations. This answer choice sounds good for now. We can eliminate answer choices A and B. Both are psychological theories that aren’t applicable here. They’re out of scope. 
  4. Psychoanalytic theory. This has to do with unconscious motivation. In our example we’re not seeing a relationship between unconscious and conscious motivation. We’re seeing community values motivate individuals to stop smoking and using tobacco. The best answer choice here is answer choice C: Incentive theory.

42) This is similar to our last question in that we’re matching something from the passage to one of our content terms.

This whole passage revolved around the idea that researchers are calling for studies to supplement the biomedical approach to cancer. That includes studies that incorporate the social determinants of disease, specifically. For this question, we want a perspective that, within the context of the passage, would satisfy these researchers. In other words, something that considers factors outside of the biomedical approach, and specifically social factors.

We’ll do the same thing we did for question 41. We’ll define the 4 key terms, and we’ll find the best answer. You’ve probably noticed for these types of questions, two or possibly even three of the answer choices are out of scope. They’re vocab terms we’re expected to know, but they’re just not applicable to the specific question. Another reminder that it’s super important to know your vocab. 

  1. Exchange-rational choice. This has to do with individuals making decisions that are rational, and people weighing the costs and benefits of actions to maximize gain. That’s not relevant in this situation.
  2. Symbolic interactionism. This has to do with using symbols to establish meanings, develop views, and communicate with other individuals. For example, seeing the word doctor spelled out is just a bunch of letters. But when you see the letters, you think of a physician that’s possibly dealing with patients. Again, not relevant in this situation
  3. Social constructionism. This has to do with interactions between individuals and the world around them giving meaning to things. Things have value, because society has decided to give them value. For example, a $100 bill is cheap to make, but we’ve decided to give it a much greater value. Social constructionism isn’t relevant in this question.
  4. Social epidemiology. This considers social factors that were explicitly mentioned in the passage. Factors like race, ethnicity, or socioeconomic status. It studies the social determinants of health, which is exactly what we’re looking for. This is going to be our best answer, we can eliminate answer choices A-C, they were all out of scope. We’re left with our best answer, answer choice D: Social epidemiology. 

43) To answer this question, we can focus on the paragraph that talks about culturally competent care. Then we’ll relate the excerpt from the passage to our vocab and the four answers given. 

Above we have the last paragraph in our passage. The focus was on culture and patient-provider interactions. It says To deliver culturally competent care, providers need to be aware of their own cultural standards and biases while understanding that patients may have different cultural standards and biases. The passage is advocating for providers to be aware of their own standards and biases, but understanding the standards and biases of their patients. 

  1. social support rather than social stigma. Social support has to do with support in the form of friends, family, and acquaintances. That can mean advice, guidance, encouragement, and emotional comfort. Social stigma are negative perceptions and assumptions against a person or characteristic. This contrast isn’t applicable to the differences in standards and biases in the passage. 
  2. cultural relativism rather than ethnocentrism. Cultural relativism is the ability to understand a culture on its own terms and not to make judgments using the standards of one’s own culture. That’s what the author mentions needs to happen from the providers. Ethnocentrism has to do with evaluating other cultures using preconceptions from your own culture. That’s exactly what providers shouldn’t be doing. We like this answer choice and can eliminate answer choice A, that was out of scope. 
  3. symbolic culture rather than material culture. Symbolic culture has to do with ideas, beliefs, values, or norms. So non-material things. Material has to do with artifacts and social relations. The description in the passage isn’t advocating for this distinction. We can eliminate this answer choice, it’s also out of scope.
  4. the front stage self rather than the back stage self. This is a theater metaphor. Front stage self refers to performing in front of an audience. In other words, actions when you’re being watched or judged by others or society. Backstage self is when no audience is present, so players can relax and not have to worry about conventions needed for the front stage self. This is also not relevant in this case. We can eliminate this answer choice, it’s out of scope. Answer choice B remains our best answer. 

44) Once again, we’re putting a name to a concept. This all boils down to vocab. The author mentions the knowledge, skills, and education required to practice medicine are associated with high social status. We want a term that describes this social status and cultural competence.

  1. The social capital of physicians. Social capital is the relationships of people who live and work in a particular society. This enables society to function effectively. This doesn’t have to do with knowledge, skills, and education required for practicing medicine. This answer choice is out of scope. 
  2. The hidden curriculum of medicine. Hidden curriculum has to do with unspoken aims of education, like having children conform to social expectations. The knowledge, skills, and education are all part of the explicit curriculum, not the hidden curriculum. This contradicts our breakdown. 
  3. The cultural capital of physicians. Cultural capital has to do with the knowledge, behaviors, and skills a person can use to demonstrate cultural competence and social status. That’s exactly what we’re looking for in this question. We can eliminate answer choices A and B because both were out of scope. 
  4. The ascribed status of medicine. Ascribed status has to do with the social status a person is assigned at birth, or assumed involuntarily. I usually don’t have to remind premed students that the knowledge, skills, and education required for practicing medicine are earned! This answer choice contradicts our breakdown. We can eliminate this answer choice. We’re left with our correct answer, answer choice C: The cultural capital of physicians.

45) This is going to be similar to question 44. To answer this question, we’re looking at the hypothetical example at the end of the passage and we’re assigning a psychology term that represents that example. 

Here we have the last part of our passage. We can focus on the example. It says For example, family members may enact cultural preferences when asking a provider to withhold aspects of a cancer diagnosis from a patient. If the provider fulfills this request, he or she may struggle to reconcile that action with norms favoring disclosure.

We want a psychological process that explains this struggle felt by the provider. There’s a struggle because the cultural preferences of the family members conflict with the provider’s existing beliefs. That’s a classic example of cognitive dissonance. We want something that either mentions cognitive dissonance, or something similar.

  1. Cognitive dissonance. This matches our breakdown exactly. We broke this question down and said the example in the passage was a classic example of cognitive dissonance. Let’s go through our other answer choices just in case we find a superior answer.
  2. A self-fulfilling prophecy. This is a phenomenon in which the expectations of a person or event unknowingly influence outcomes. This ultimately leads to the realization of those expectations. Not the case in this situation and this answer choice is out of scope. We can eliminate answer choice B.
  3. Confirmation bias. Confirmation bias has to do with the tendency to look for information that supports your idea or approach instead of information that may contradict your approach. Not what’s happening here, despite the contradiction in beliefs. We can eliminate this answer choice, it’s also out of scope. 
  4. The fundamental attribution error. This has to do with overemphasizing internal factors as explanations for the behavior of the other person. So, we assume actions are the results of traits, and we downplay the role of context. Not the case in this question. This is also out of scope. We can eliminate answer choice D. We’re left with our correct answer, answer choice A: cognitive dissonance. 

46) Unlike the previous questions related to this passage, this time we’re given a topic from AAMC’s content outline and we have to relate a topic from the passage that’s most consistent with the topic. We’re looking at social cognitive theory and modeling. 

Social cognitive theory says that portions of an individual’s knowledge acquisition can be directly related to observing others. Modeling occurs from watching, retaining, and replicating a behavior observed from a model. So, when people observe a model performing a behavior, they can use that information to guide their own behaviors. In the passage, the big example that sticks out is children observing health behaviors inconsistent with smoking. Eventually that leads to establishing norms against tobacco use in that community. We’ll look at our answer choices and find one consistent with the emphasis on modeling.

  1. Smoking prevention by establishing norms against tobacco use. This is consistent with social cognitive theory’s emphasis on modeling. In fact, this was the exact example I used in the breakdown. Children and young adults learn to not smoke or use tobacco by observing health behaviors inconsistent with smoking. These children replicate that behavior. It’s the best example in the passage of modeling, so we like this answer choice. This question essentially just comes down to knowing the vocabulary, but we’ll still look at the other 3 options.
  2. Understanding the treatment implications of cultural differences. Social cognitive theory has to do with learning behaviors and modeling. Understanding treatment implications of cultural differences is important, but it doesn’t have to do with observing, retaining, and replicating behavior. We can eliminate this answer choice because it’s out of scope.
  3. Obtaining the skills and education required to practice medicine. You guys can attest to this, a lot of the education and skills required to practice medicine come from reading and understanding information, and not only modeling. Granted there is a level of modeling later in your career, but that’s only part of the entire equation. Answer choice A is still a superior answer choice here, it’s the most consistent with modeling. We can eliminate answer choice C.
  4. Supplementing the biomedical approach with other perspectives. Another good practice, but not something that has to do with modeling. Supplementing the biomedical approach with other perspectives can lead to better health behaviors and patient-provider interactions, but it’s not as consistent with modeling or social cognitive theory. We can eliminate this answer choice because it’s out of scope. We’re left with our correct answer, answer choice A. 

Section Bank: Psychological, Social, and Biological Foundations of Behavior: Questions 47-51

47) This is a standalone question that is focusing on a key term. To answer this question, we can consider what we know about intersectionality. Intersectionality is key when understanding social class. All individuals have multiple aspects of identity, and simultaneously experience some privileges due to their socially valued identity statuses and disadvantages due to their devalued identity statuses. The intersection of these aspects is what’s key to understanding intersectionality. 

  1. micro and macro levels of analysis. Macro level looks at larger-scale social processes, while micro level looks at smaller-scale interactions between individuals. We’re focused on individuals having multiple aspects of identity.
  2. symbolic and material culture. Material culture refers to the relationship between artifacts and social relations while symbolic (or nonmaterial) culture refers to the ideas, beliefs, values, or norms that shape a society. This is going to be similar to answer choice A. this is not something we can really narrow down and pick out the multiple aspects of one’s identity.
  3. race/ethnicity and social class. This sounds like the perfect example for what we said in our breakdown of the question. A person’s place in society is affected by not only race/ethnicity, but also by social class. It doesn’t always have to be these two categories, but this is a viable example. Answer choice C is going to be the best option so far. 
  4. cultural values and social norms. Social norms are the explicit or implicit rules specifying what behaviors are acceptable within a society or group and include sanctions, folkways, mores, taboos, and anomie. Cultural values include culturally-defined standards that serve as broad guidelines for social living. Neither one of these has to do with intersectionality and a person’s identity. When we thing intersectionality, we’re thinking race/ethnicity, age, sex, or social class. We can stick with answer choice C as our best answer.

48) As with many behavioral standalone questions, we can answer this question by knowing our vocabulary. We’re given a description and we have to match that description to one of the four terms in the answer choices. Let’s find the one that best matches the question stem.

  1. Functionalism this is a framework for building theory that sees society as a complex system whose parts work together to promote solidarity and stability.
  2. Exchange-rational choice This choice assumes that every action is fundamentally rational, and the person acts by weighing the costs and benefits of each action to maximize their personal gain
  3. Conflict theory this is a way of studying society by focusing on inequality between different groups and sees social life as a competition and focuses on the distribution of resources and power between these groups
  4. Symbolic interactionism this focuses on small scale perspectives with small interactions between individuals. First part of this definition is finally consistent with what we’re focused on in the question stem. We’re looking at the micro level. Social interactionism explains how individuals act in society and can be expanded to look at the interactions of larger social groups to explain social change. The healthcare providers and patients are the individuals, and their interactions are influenced by how the healthcare providers establish rapport, show empathy, and navigate disagreements. This is our best answer choice.

49) Similar to our last question and most of the standalone questions in this section, we’re relying on knowing our vocabulary. We can define the concept in the question stem, then see which is most likely for a person who acquires a stigmatized illness. 

The looking-glass self states people see themselves based on how they believe others perceive them during social interactions. There are three main components of the looking glass self: we imagine how we must appear to others, we imagine the judgment of that appearance, we develop our self through the judgments of others

  1. The person will seek out other people with the same stigmatized illness. This may or may not happen, but remember we’re looking for a reaction that’s most likely and within the context of the concept of the looking-glass self. This is not likely
  2. The person will internalize the perceived stigmatization against him or her. This sounds like a great answer. If society stigmatizes this illness, the person will also internalize the perceived stigmatization. Why is that? We imagine how we must appear to others, we imagine the judgment of that appearance, we develop our self through the judgments of others. This is a good answer for now.
  3. The person will redirect feelings of stigmatization toward out-group members. In sociology and social psychology, out-groups are social groups towards which an individual feels contempt, opposition, or a desire to compete. While there are negative feeling, we’re not redirecting feelings of stigmatization. Remember, we’re staying within the context of the concept of the looking-glass self! Answer choice B remains the best option for now.
  4. The person will become stigmatized by his or her secondary group members. Secondary groups are large groups whose relationships are impersonal and goal-oriented; their relationships are temporary. Again, something that may or may not happen, but we’re thinking within the context of the concept of the looking-glass self. This is incongruent with the main components of the looking-glass self so we can eliminate this answer choice. Answer choice B is our best option.

50) Most behavioral standalone questions come down to knowing external knowledge. However, this question comes down to reading and interpreting the population period in the question stem. We can note that population (in millions) increases until higher ages (60+) before leveling off. There are more relatively older people in this population and the “base of the pyramid” is smaller. We can see if that plays a role in the correct answer.

  1. The overall size of the population is likely to decrease. Considering there are so many older people in the population, we actually expect the population to decrease soon. The base of the pyramid (younger people) is much smaller, so population will decrease when the older people die.
  2. The mortality rate of the population is unlikely to change. This may or may not happen, but it’s not going to be the best supported by the image in the question stem. The mortality rate can remain the same, or change significantly. That’s not what we’ll gather form this figure.
  3. The median age of the population is unlikely to change. This is false. The median age will likely decrease as the older generation dies. We talked about this with answer A, also. The base of the pyramid (younger people) is much smaller. When the larger, older group dies, the median age should decrease.
  4. The birth rate of the population is likely to increase. There is no way to say if this is true or false. We actually expect fewer births to happen simply because there are fewer young people in the population. This is inconsistent with the visual so we can eliminate answer choice D as well. Answer choice A is going to remain our best answer. 

51) This is a vocab question. Demographic transition theory: suggests that future population growth will develop along with a predictable four-stage model. In Stage 1, birth, death, and infant mortality rates are all high, while life expectancy is short. As countries begin to industrialize, they enter Stage 2, where birthrates are higher while infant mortality and the death rates drop. Life expectancy also increases. Stage 3 occurs once a society is thoroughly industrialized; birthrates decline, while life expectancy continues to increase. Death rates continue to decrease. In the final phase, Stage 4, we see the postindustrial era of society. Birth and death rates are low, people are healthier and live longer, and society enters a phase of population stability.

Which psychological theory best explains the role of community values in the preliminary studies of tobacco cessation?

a.

Which psychological theory best explains the role of community values in the preliminary studies of tobacco cessation?

Based on our breakdown and the visual I provided, this is exactly what we’d expect. Both birth rate and death rate decrease (with death rate decreasing first and population size growing). b. 

Which psychological theory best explains the role of community values in the preliminary studies of tobacco cessation?

While death rate is doing what we expect, we also expect birth rate eventually decreases and the population will stabilize. Answer choice A remains superior. c,

Which psychological theory best explains the role of community values in the preliminary studies of tobacco cessation?

This is the opposite of what we expect. Birth and death rate will both, eventually decrease, not increase. Answer choice A remains superior. d.

Which psychological theory best explains the role of community values in the preliminary studies of tobacco cessation?

We expect birth rate to decease over time, the same as death rate. Once again, we can eliminate this answer choice. Answer choice A is our correct answer.  

Section Bank: Psychological, Social, and Biological Foundations of Behavior: Passage 7

52) To answer this question, we’ll have to identify the technique synonymous with measuring brain activity. In the passage, the author alluded to fact that the researchers compared brain activation in response to images of food in two scenarios: after adequate sleep, and when sleep deprived. The breakdown of this question comes down to knowing your content and picking the technique that fits our criteria. Safely measuring increased neural activation of specific brain regions would happen by functional MRI. fMRI is a common type of brain scan. It’s specifically used to measure oxygen levels in specific areas of the brain, as a measurement of brain activity. It’s used to detect changes associated with blood flow. We’ll try and find an answer choice that either mentions fMRI, or something with a similar function.

  1. MRI. MRI is partially correct, we at looking at specific brain regions. But this question specifically wants to know how we measure increased neural activation in specific brain regions. fMRI is different from MRI because it’ll actually show this activity, and not just the anatomical structure.
  2. fMRI. This answer choice matches our breakdown exactly. We’re focused on a technique that can specifically measure neural activity. That’s exactly what happens when using fMRI. You can detect differences in blood flow using fMRI. Both MRI and fMRI will you show you structures. But answer choice B is superior here because it’ll show you he increased neural activation of the specific brain regions. We can now eliminate answer choice A.
  3. CT. CT scans are used more for bone and joint problems. CT scans use X-rays to produce images. They can show you internal structures, but we’re keeping our eyes on the prize here. CT scans won’t show the increased neural activation which is what we’re looking for. We can eliminate this answer choice because it doesn’t fulfill our requirement.
  4. PET. PET stands for positron emission tomography. Gamma rays are used to mark activity, which can be helpful if you’re looking for something like cancer. A scan can help with measuring activity in general, but it wouldn’t be able to fulfill the specific brain regions criteria. We can eliminate this answer choice. We’re left with our correct answer, answer choice B: fMRI.

53) This answer to this question is going to come from analyzing the findings in the passage, then defining the relationship between inadequate sleep and hunger. We’ll focus on defining relationships between variables. In the study, individuals are deprived of sleep, and then asked about their hunger levels. The people that did not sleep were hungrier (they had higher hunger ratings). The ones that slept normally were less hungry, meaning sleep deprivation caused the hunger.

  1. There is a direct cause and effect relationship between sleep and increased feelings of hunger. A direct cause, like the name suggests, would mean sleep directly affects hunger. In this scenario, sleeping decreases hunger, while sleep deprivation reduces hunger. Based on the study results, the sleep deprivation actually affected different parts of the brain. That, in turn, caused the higher hunger ratings and negative feelings overall. This is more of an indirect cause and effect, but we’ll still keep this answer choices.
  2. There is a positive correlation between hunger and inadequate sleep. This implies either sleep deprivation causes hunger or hunger causes sleep deprivation. We know the first is true, but not the second. We don’t have evidence that hunger causes sleep deprivation. We can eliminate this answer choice because it’s factually incorrect.
  3. There is evidence that inadequate sleep causes hunger. Thinking back to our breakdown, this answer choice is reasonable because of the way the study was conducted, and how the results were presented. We said researchers found that the people deprived of sleep were hungrier. How did they find this out? By conducting an experiment. They varied the independent variable, the sleep, and they tracked the dependent variable, the hunger ratings. The results from the experiment are evidence that sleep deprivation causes hunger. We already mentioned we can’t say with certainty that there is a direct cause and effect relationship, so we can eliminate answer choice A.
  4. There is a negative correlation between hunger and inadequate sleep. This is similar to answer choice B. We don’t have a correlation between these two variables. If we did, then hunger would influence sleep. That may or may not be true, but it’s certainly not something we can figure out using these study results. We can eliminate answer choice D. We’re left with our correct answer, answer choice C: There is evidence that inadequate sleep causes hunger

54) There’s no real way around answering this question. We’ll have to know the sleep cycles and stages, and know which one is characterized by sleep spindles. The question stem mentions brain wave activity was recorded, but we shouldn’t need brain wave information to answer this specific question. 

4 stages of sleep. We have non-rapid eye movement (NREM) stage 1. This is the initial transition between wakefulness and sleep.

NREM stage 2 is when temperature drops and heart rate slows. This is when the brain produces sleep spindles. We’re going to look for NREM stage 2 in our answer choices.

Just to be thorough: NREM stage 3 is when muscles relax. Blood pressure and breathing rates drop.

In REM sleep, the brain is more active, the body is relaxed, eyes move rapidly, and dreams occur. A person completes the entire sleep cycle about four or five times in a given night of full sleep.

We know exactly the answer we’re looking for in this case. We can eliminate answer choices A, C, and D for being incorrect. We went through the different stages and said the stage characterized by sleep spindles is answer choice B: NREM stage 2.

55) The only detail we need to know from the passage is the children that are targeted by advertisements are 5-7 years old. All we’re going to do is classify this age range within Piaget’s stages of cognitive development.

  1. Formal operational. This corresponds to an age range of 11 years and older. This is when we see theoretical and hypothetical thinking. There’s abstract thoughts and reasoning, strategy, and planning. But this is older than what we’re looking for in this situation.
  2. Sensorimotor. This corresponds to ages 0-2 years old, sometimes 0-1. It involves coordinate of senses with motor responses. We see object permanence. This is too young for what we’re looking for. 
  3. Preoperational. This corresponds to ages 2-7. Children start to talk and use symbolic thinking. This is exactly the range we’re looking for. We can eliminate answer choices A and B, both contradict what the author says in the passage about the children’s ages.
  4. Concrete operational. This corresponds to ages 7-11 years old. Like the name suggests, there’s now concrete thought and logical reasoning. This is slightly older than the target age group in the passage so we can eliminate answer choice D. We’re left with our correct answer, answer choice C: Preoperational. 

56) We expect similar activation levels in three of the brain regions listed, and we want to find the brain region in which we’d be LEAST likely to find similar activation levels. The answer is going to come from knowing which brain regions would be activated when viewing the images of food.

  1. Nucleus accumbens. This has to do with reward and reinforcement so we do expect activity here. 
  2. Hypothalamus. Hypothalamus has to do with hormone release, but also controls hunger, so we expect activity here also. We have some level of activation in answer choices A and B, let’s keep comparing.
  3. Cerebellum. Cerebellum has to do more with motor control, not with hunger or reward and reinforcement. So far, this is the odd-answer choice out for now.
  4. Amgydala. Amygdala has to do with emotional learning and is also part of the reward pathway. We have 3 of our 4 answer choices here where we’d see activity. We can eliminate answer choices A, B, and D. We’re left with our correct answer, answer choice C. We’d be least likely to find similar activation levels in the cerebellum.

57) The answer to this question is going to come from AAMC’s content outline. “motivation” is a topic under content category 7A. Motivation can come from instinct, drives, or needs. We’re focused on which aspect of the study will be most relevant to influencing motivation from an evolutionary perspective.

  1. Hunger ratings for images of food. This is consistent with our breakdown of the question. Hunger is a basic biological drive, which we mentioned will influence motivation. The images were specifically of high-caloric foods, which humans evolutionarily crave. Think about why the sugariest and the fattiest foods taste the best to a lot of people. If we don’t have a lot of resources, that’s the food that will help us survive longest. Researchers would be interested in these ratings. 
  2. Fasting plasma glucose concentrations. This is dealing more with actual physiology, not as much motivation. Remember our focus here is on evolutionary perspective of human motivation. We can eliminate this answer choice.
  3. Activity in the anterior cingulate cortex. This is similar to answer choice. B. Brain activity is dealing more with physiology, and not as much the actual motivation. We can also eliminate this answer choice.
  4. Negative and positive affects. This is dealing with the emotions felt by the individual participants, not specifically motivation. Researchers are focused on an evolutionary perspective of human motivation. That corresponds to the hunger ratings in response to the images of food. We’re left with answer choice A as our best option. 

Section Bank: Psychological, Social, and Biological Foundations of Behavior: Passage 8

58) To answer this question, think about how in Study 1 infants were shown faces with universally expressed emotions. 3 of the answer choices will be universally expressed emotions, one will not. We’re looking for the odd one out here. This is almost like a pseudo-discrete or standalone question. 

Which psychological theory best explains the role of community values in the preliminary studies of tobacco cessation?

We have the common universal emotions associated with distinct facial expressions. That includes happiness, surprise, sadness, fear or fright, disgust, contempt, and anger. Keep in mind that you might have synonyms for some of these in the answer choices, but all we’re doing is finding an answer choice that’s not on this list.

  1. fear. This answer choice is one of the universal emotions we went through and is shown in the visual.
  2. surprise. This is also one of the universal emotions we went through.
  3. pain. Pain was NOT one of the emotions listed, so this answer choice stands out for now. We can eliminate answer choices A and B because those contradicted our visual and what we know from our content.
  4. disgust. This is another answer choice that was one of the universal emotions we listed. We can eliminate this answer choice as well because it goes against our content. We’re left with our correct answer, answer choice C: Pain.

59) In other words, we want to explain why 9-month old infants’ recognition was restricted to own-race faces, but 6-month old infants could recognize own-race and one of the other-race faces. This is very similar to question 58 as it is almost like a pseudo-discrete or standalone question. We’re going to explain the difference in age, and why there are different results when recognizing faces, and attachment.

9-month old infants’ recognition was restricted to own-race faces. That’s likely due to a stronger attachment to own-race faces. Newborn babies up until about 6 weeks old will only form the beginnings of attachment. Complete attachment hasn’t happened yet, and the baby is okay with being left with unfamiliar people.

Around 6 weeks to 8 months, that’s when we go through the attachment-in the making phase. Like the name suggests, attachment is getting stronger. Infants will start responding different to familiar people than they do with strangers. There’s no separation anxiety yet. Attachment is being cultivated. This is where we have our 6-month olds in the study.

From 8 months to 18 months, we have clear-cut attachment. Attachment is strengthening and toddlers will stick to their caregivers and the adults in their lives. This is what we’re seeing in the 9-month old infants.

Ultimately, it comes down to the attachment these infants feel to familiar faces as they grow older.

  1. Stranger anxiety. This answer choice is consistent with our breakdown. We mentioned separation anxiety doesn’t really form until 8 months, and the 6-month old infants aren’t as attached to familiar faces as the 9-month old infants. The 6-month olds are still building attachment. This also explains why these infants were able to recognize only one of the other-race faces, but the 3-month old infants recognized all three other-race faces. The 3-month olds hadn’t developed that same attachment.
  2. Critical period. This answer choice is a vocab term, but has more to do with language acquisition. This is out of scope and not answering the specific question being asked.
  3. Object permanence. This answer choice tefers to the idea that objects exist outside of our senses-meaning when we can’t see or hear or smell them. That’s something that develops in young children, but again something that’s not relevant to what we’re looking for here.
  4. Autonomy. Autonomy usually comes later for toddlers, but it involves learning they can control their actions and their actions get results. Again, not relevant to the results in Study 1. We can eliminate answer choice D. We can pick the only answer that explains the difference in results in Study 1: answer choice A-Stranger Anxiety

60) Be careful with the verbiage here, we want a concept that does not explain the ability of participants to visually process faces. This is 3 straight questions in this set where we’re tying the question back to the passage, but the answer comes from knowing your content. We’re focusing on visually processing new faces in both studies.

  1. Parallel processing. Parallel processing is the use of multiple pathways to convey information about the same stimulus. For visual processing that involves putting together pieces of information to create a final, visual picture. 
  2. Interposition. Interposition also has to deal with visually processing faces, and more with depth. If you see objects overlapping, you can perceive depth. Even though you might not see something that’s being covered up, you can still tell the object that’s being covered up is further away. 
  3. Accommodation. Accommodation has to do with revising perceptions and current understanding based on new information. This new information has to be incorporated, or accommodated. This doesn’t exclusively have to do with visual information, but it is relevant here. That would make this the frontrunner for now because answer choices A and B were more focused on visual processing specifically.
  4. Place theory. Place theory has more to do with sound and perceiving sound. This has more to do with frequencies producing vibrations along the basilar membrane, not visually processing faces. Luckily for us, that’s exactly what we’re looking for in this question. This option does not explain the ability of the participants to visually process the faces they were shown. We can eliminate answer choices A-C. We went through their definitions and reasoned how each one does help process the faces. That means those answers contradict what we’re looking for in this question.

61) To answer this question, we can go back to Study 2 to get details about the brain imaging technique. We’ll likely have to us our general knowledge to explain the technique, within the parameters of MCAT content.

We’re focusing our attention to the middle of the passage. It says “Adult participants viewed other-race and own-race faces while their brain activity was measured using functional magnetic resonance imaging.” What imaging technique are we focused on? Functional magnetic resonance imaging, or fMRI. fMRI is a common type of brain scan. It’s used to measure oxygen levels in specific areas of the brain, as a measurement of brain activity. It can be used to detect changes associated with blood flow. So we want something along the lines of measuring brain activity, detecting changes associated with blood flow, or measuring oxygen levels.

  1. measures increased levels of glucose. We know this answer choice contradicts our breakdown. fMRI focuses on measuring brain activity, detecting changes associated with blood flow, and measuring oxygen levels. Let’s see if we can get a better answer choice. 
  2. records increases in electrical activity. This isn’t quite correct either. fMRI could reveal areas of increased blood flow or brain activity. But not electrical activity. So far, neither A nor B are what we’re looking for. 
  3. reveals areas of increased blood flow. Bingo! This is what we want from our correct answer. We said when describing fMRI, we talk about measuring brain activity and detecting changes associated with blood flow. This answer choice is consistent with our breakdown, so we can now eliminate answer choices A and B.
  4. measures increases in neurotransmitter levels. This is not what’s being measured in fMRI. No need to dwell on this answer, it also contradicts the definition in our breakdown. We can eliminate answer choice D. We’re left with our correct answer, answer choice C.

62) This is a very open-ended question, so we’re going to attack this by keeping in mind the key points from Study 2.

Quick overview of Study 2: Participants viewed various combinations of faces for different amounts of time. fMRI was used to measure brain activity. With the quicker exposure, we get greater activation in the amygdala for the other-race faces. With the longer exposure, that difference is reduced, but other-race faces activate the frontal cortex more. These are the results we’re given, but now we’re asked to find an applicable scenario. 

  1. is fearful of contact with the stranger if experienced in a new environment. This may or may not be true, but we want to know which scenario represents the results of study 2, meaning within the context of this passage. The author does mention when faces were presented for 30 milliseconds, there’s activation of the amygdala. The amygdala is a brain region that responds to fear, but there’s no mention of the effect of a new environment.
  2. initially displays a frown, and then proceeds to greet the stranger. Again, something that may or may not happen, but this sounds like a better answer than answer choice A. Why is that? The author mentions an automatic response is followed by a conscious and controlled cognitive process. We don’t know what that result will be, but like we said, this matches our criteria better than answer choice A, so we can eliminate A and keep answer choice B. 
  3. mentally compares the stranger to family members, which gives them a feeling of comfort. This is similar to answer choice A where this may or may not happen, but there’s no mention of this in the passage, and we don’t have enough information in the question stem to assume this happens. We can eliminate answer choice C as well. B is still superior.
  4. experiences a brief increase in heart rate before starting to relax. This is certainly a possibility. We said when faces were presented for 30 milliseconds, there’s activation of the amygdala. The amygdala is a brain region that responds to fear, which explains the brief increase in heart rate. The longer exposure leads to this effect being reduced, or subsequent relaxation. This sounds like the most likely answer choice in this situation. B was a decent option, but we don’t know if there will be a greeting of the stranger or not. That’s making a connection that we weren’t really given in the passage. We can eliminate answer choice B. We’re left with our correct answer, answer choice D.

63) To answer this question, we’re describing the correlation between age and recognizing emotions on other-race faces in infants from ages 3, 6, and 9 months old. We can revisit the point in the passage where the author explained the recognition for own-race and other-race faces for all three ages.

From the passage: the results showed 3-month-old infants demonstrated recognition for own-race and all three other-race faces, the 6-month-old infants were able to recognize own-race and one of the other-race faces, and the 9-month-old infants’ recognition was restricted to own-race faces. 

What does that sound like? In these particular age groups, as age increases, we have a decrease in the ability to recognize emotions on other-race faces. That’s a negative, or inverse correlation.

  1. positive. This answer choice contradicts our breakdown. A positive correlation would mean the older infants would demonstrate recognition for more other-race faces. The opposite is true.
  2. negative. This is consistent with our breakdown, and it’s the opposite of answer choice A. We can eliminate answer choice A. As age increases, we have a decrease in the ability to recognize emotions on other-race faces. Meaning we have a negative correlation.
  3. zero. This answer choice also contradicts our breakdown. There is a clear correlation between age and the ability to recognize emotions on other-race facts. We can eliminate this answer choice. 
  4. curvilinear. A curvilinear relationship will change direction after a certain point. Two variables might be increasing at the same time, but after a certain point, one variable will start decreasing. That’s not what we’re seeing in this specific sample. This might be true in a different sample, but for the same of Study 1, we have a negative correlation. We can eliminate answer choice D. We’re left with our correct answer, answer choice B: negative. 

Section Bank: Psychological, Social, and Biological Foundations of Behavior: Questions 64-67

64) We’re mixing some sciences here! This is actually closer to what you’ll see on the exam where you’re expected to have a base level of general knowledge that you can apply across disciplines. Even though this is still a behavioral question, it does require a basic level of understanding of action potentials which we normally see in the biological and physical sciences. An action potential is a short-term change in the electrical potential that travels along the cell. We want to know which of the four situations listed in the answer choices would stimulate an action potential. This is a content question.

Which psychological theory best explains the role of community values in the preliminary studies of tobacco cessation?

  1. In any excitable cell, if neurotransmitter binding opens ligand-gated channels at the synapse. We can see this in our figure above, but we don’t know exactly which channels would open. We specifically want ligand gated cation channels open. This is not specific enough. 
  2. In a postsynaptic neuron where a depolarizing change in membrane potential exceeds threshold. Once again, looking at our figure, once the depolarization stimulus exceeds our dotted line (threshold), we have our action potential. This describes exactly what we need and expect to happen. This is our best answer choice so far.
  3. In a postsynaptic neuron that expresses a high density of neurotransmitter receptors. We need to make sure these receptors are the specific ones we need, and not just any receptors. We could have anion channels open, which wouldn’t be the same effect. This is similar to answer choice A in that it’s not as specific as we’d like. Answer choice B remains superior.
  4. In any excitable cell, if the neurotransmitter released into the synaptic cleft is acetylcholine. This is once again not as specific as we’d need. While acetylcholine is going to be the neurotransmitter in an action potential, it can have different effects in different situations. Answer choice B is going to be the best answer choice 

65) This is a standalone question that relies on knowing the content. We’re going to focus on Content Category 6A which contains the subject “Vision.” If you want to review vision, make sure to check our content outline: https://jackwestin.com/resources/mcat-content/vision. Specifically, we’re going to consider the fovea in this situation. The night sky example is one AAMC likes to use. From our content outline: The fovea is the region in the center back of the eye that is responsible for acute vision. The fovea has a high density of cones. When you bring your gaze to an object to examine it intently in bright light, the eyes orient so that the object’s image falls on the fovea. However, when looking at a star in the night sky or other object in dim light, the object can be better viewed by the peripheral vision because it is the rods at the edges of the retina, rather than the cones at the center, that operate better in low light. In humans, cones far outnumber rods in the fovea. 

  1. away from the blind spot where no photoreceptors are present. If the star were in her blind spot, she would not have seen it when she initially looks up at the night sky. This is not a good option.
  2. to a region of the retina where photoreceptors have a higher threshold for light detection. If there were a higher threshold for light detection, she would not have been able to see the star as easily. Neither answer choice A nor answer choice B are great options. 
  3. away from the fovea toward the periphery of the retina. This matches what we said in our breakdown of the question. I mentioned when looking at a star in the night sky or other object in dim light, the object can be better viewed by the peripheral vision because it is the rods at the edges of the retina, rather than the cones at the center, that operate better in low light. In humans, cones far outnumber rods in the fovea.
  4. to a region of the eye where photoreceptors contain more than one type of retinal pigment. This answer choice is out of scope. Additional types of retinal pigment are not the reason for the reappearance of the dim star in her vision. We can stick with answer choice C as our best option here.

66) At first glance this might look similar to question 65 which tests vision, but the test-maker takes a turn in the second and third sentences here. We’re focused instead on the scores of the tests of visual perception, and why they differed on consecutive days. This question, like many questions in the behavioral section, will come down to knowing our vocab. We’ll break down the four options and pick the one that would explain the shortcoming of the test in question. 

  1. valid. If a test is valid, it’s giving you results based on what you’re looking for specifically. What do I mean by that? In this case, the researchers wanted to test visual perception, and they managed to get results for two days in row. Just because the scores were different, does not mean the test is not valid. 
  2. standardized. The test in question may or may not have been standardized, but there’s nothing in the question stem that suggests the test is not standardized. That’s the key here. What is suggested by the different scores? Theoretically, these students were all tested in the same, or a similar way.
  3. reliable. This is exactly what we’re looking for in an answer choice. If we had a reliable test, we’d expect consistent results from one day to the next. The fact that the scores varied so much in just one day shows the test is not reliable. This is going to be the best answer choice so far.
  4. generalizable. A generalizable test would be able to be used on a larger population. If we have a group of students, the results should be able to be applied to a similar group as this studied group of students. That’s not the issue presented in the question stem. 

67) Intelligence involves the ability to adapt to one’s environment and the capacity to learn from experience. There are a lot of different theories and ideas around intelligence. For example, Howard Gardner published a book on multiple intelligence that breaks intelligence down into at least eight different modalities: logical, linguistic, spatial, musical, kinesthetic, naturalist, interpersonal, and intrapersonal intelligence. Looking at our specific example, we see a +0.38 correlation. A correlation of 0 would imply there’s no relationship between the score on the two tests. A correlation of 1 would imply that the performance on the verbal test would predict performance on the spatial test. In this case, we have some overlap between the tests. It’s not quite 1, but it’s also not a correlation of 0. What is the g factor? It’s a measure of general intelligence. We have both of our tests that measure types of intelligence, but general intelligence is made up of different cognitive abilities. A high g factor likely means decently high scores on any intelligence test, despite the correlation being less than 1. 

  1. performance on both tests is partly determined by “g.” This is consistent with what we know about the general intelligence factor. General intelligence does not necessarily only include verbal intelligence or spatial intelligence. However, it’s reasonable that performance on intelligence tests will be partly determined by general intelligence factor. We like this answer choice.
  2. the two tests measure different things, which does not include a “g” component. Even though the tests measure different things, they are both intelligence tests. We assume general intelligence is needed to score decently on these intelligence tests, so they should include a “g” component. 
  3. both of the tests are excellent measures of “g.” This can’t be effectively concluded. There is a positive correlation between the two tests, but we can’t say for sure they are excellent measures of “g”. If we had a higher correlation and we knew how much each test influences “g”, then we could look further into this conclusion.
  4. that “g” is a statistical artifact with no real-world validity. General intelligence is a helpful measure that correlates to things like verbal intelligence and spatial intelligence. I measured the 8 modalities Gardner published in his book. General intelligence would theoretically positively influence most of those modalities. We can eliminate this answer choice. Best option here is going to be answer choice A. 

Section Bank: Psychological, Social, and Biological Foundations of Behavior: Passage 9

68) Be extra careful with the verbiage. Presentation of the WAIS was counterbalanced for specific reasons. 3 of the answer choices will be legitimate justifications for doing so, but the one that’s not a justified reason is going to be our correct answer. Fluid intelligence was measured with the WAIS, but the presentation was counterbalanced. Half the participants completed the WAIS before the cognitive tasks, and the other half completed it after the cognitive tasks. Counterbalancing is good experimental procedure because it minimizes the effect of the order influencing the results. It spreads order effects evenly across the two tasks in this situation.

  1. Participants may become fatigued causing performance to decline as the experiment progresses. This is a justification for counterbalancing and one that gets brought up a lot. The researchers can spread the effects of this fatigue evenly by counterbalancing. Any decline in performance can be spread among both tasks. 
  2. Some tasks may be more exciting or boring than others and may affect how participants respond to subsequent tasks. This is again a justification for counterbalancing. Counterbalancing can address and minimize this effect because only half of the participants did one task first, while the other half did the second half. Because this is consistent with counterbalancing, we’re not liking this answer. 
  3. It is an effective method for controlling the order in which stimuli are presented. Another common justification for counterbalancing. This ties into the experimental setup for us too. Half of the participants went in one order to complete all the tasks, while the other half went in a different order. This identifies and addresses biases and issues that may appear from going in a certain order. Again, valid justification for counterbalancing.
  4. It ensures that all participants are presented with the same variables. This is something that happens regardless of counterbalancing. All of the participants went through the same tasks. Counterbalancing was just used to prevent biases that pop up when going in a certain order. It doesn’t change the variables. That makes this the odd-choice out. The one that best answers our question. We can eliminate answer choices A-C, those all contradicted what we were asked in the question stem. We’re left with our correct answer, answer choice D. 

69) This question is almost like a standalone question where we don’t need information from the passage. To answer this question, we’re picking a negative symptom of schizophrenia. What are negative symptoms? Negative symptoms are conspicuous by their absence. For example, not being able to show emotions, a lack of pleasure or withdrawal, a lack of communication skills. A lack of normal function.

  1. Delusions. Delusions are a positive symptom. Positive schizophrenia symptoms mean a presence of symptoms, rather than an absence. Delusions are beliefs that are false, and indicate abnormal thinking. This is the opposite of what we’re looking for.
  2. Hallucinations. These involve seeing, feeling, and hearing things others don’t. This is another positive symptom. 
  3. Emotional flattening. The way I can describe this is a lack of emotion. No emotion in the voice or facial expressions. Little emotion in conversation or when something happens. This absence of emotion is a negative symptom. We can eliminate answer choices A and B, both were positive symptoms.
  4. Disorganized speech. This ties into not being able to organize thoughts and convey them properly. This is another positive symptom. We’re left with our correct answer, answer choice C: Emotional flattening.

70) This is another pseudo-discrete or standalone question. What is priming? It’s a change in response to a stimulus because of a subconscious memory effect. Think of it this way. If I give you a bowl of candy and ask you to pick the red candy only, you will focus on the red candy and try and block out the other colors. If you keep picking out red candies, you continue to block out the other colors. After a few rounds, if you’re asked to pick a blue candy, it takes a bit to readjust and change your reactions from blocking out the non-red candies, to now blocking out the blue candies. That’s the negative priming effect. You’re used to blocking out the blue candies when you were picking out only red candy. Now you have to shift your attention to blue candy that you previously ignored.

The author says in the passage that negative priming is slow, error-prone responses to stimuli that were previously ignored. We’ll go through our answer choices and find a type of memory consistent with negative priming.

  1. Implicit memory. These include long-term memories that are not consciously remembered, including procedural memories. This is what we’re dealing with in the passage. Changing from the red candy to a blue candy requires procedural memory, not a fact or event that’s stored.
  2. Declarative memory. This is the other type long-term memories that are consciously remembered. There are facts and events that are stored and can be consciously recalled or declared. Negative priming doesn’t involve recalling facts like this. We can eliminate this answer choice.
  3. Iconic memory has to do with visual information. Iconic has to do with recalling a picture or image. Not what’s happening in negative priming. We can eliminate this answer choice.
  4. Explicit memory which is just another word for declarative memory. Both B and D are saying the same thing. You can only have one correct answer per question on the MCAT, so if two answers are identical, we can eliminate them both. We’re left with our correct answer, answer choice A: Implicit memory.

71) Negative symptoms are conspicuous by their absence. For example, not being able to show emotions, a lack of pleasure or withdrawal, a lack of communication skills. A lack of normal function. We want to identify the class of medication that would either make these symptoms worse, or cause these symptoms to show up. In general, antipsychotic medications at used often on schizophrenic patients. These do have a sedative effect and can cause a decline in mental abilities. 

  1. Atypical antipsychotics. These have different side effects compared to antipsychotics, or typical antipsychotics. We mentioned the sedative effect from typical psychotics. Atypical antipsychotics have different known side effects. These include weight gain, metabolic problems, and sexual side effects. 
  2. Neuroleptics. These are also known as antipsychotics, and typical antipsychotics. This is consistent with our breakdown, so we can keep this answer choice for now. We can eliminate answer choice A because it contradicts our breakdown.
  3. Hallucinogens. Hallucinogens actually cause some of the symptoms in schizophrenia patients. This is not a viable medication these participants would be taking. 
  4. Stimulants. Stimulants would do the opposite of exacerbating negative symptoms. They function to raise physiological activity in the body, so there wouldn’t be a lack of emotions, pleasure, or communication skills. This contradicts our definition, we can eliminate answer choice D. We’re left with our correct answer, answer choice B: neuroleptics.

72) To answer this question we can go back to the passage where the author talks about fluid intelligence.

We have part of our passage here it says a measure of fluid intelligence was included to investigate whether any relationships between symptoms and cognitive functioning were maintained despite differences in general cognitive ability. 

During our readthrough we said fluid intelligence involves thinking, reasoning, and solving problems in novel situations. It’s not based as much on past learning and experiences, but instead on reacting, adapting, and reasoning. We also know from the passage that Fluid intelligence was significantly positively correlated with verbal fluency in the SPD group. So, there is a connection and a relationship there.

  1. positively influence performance on the cognitive tasks and should be controlled for in the study. This answer choice is consistent with what we said in our breakdown. We said fluid intelligence was positively correlated with verbal fluency in the SPD group. If the researchers didn’t assess fluid intelligence, there would’ve been no way to know that it should’ve been controlled for in the study.
  2. positively influence scores on the APNS scale and should be controlled for in the study. Fluid intelligence was positively correlated with verbal fluency in the SPD group only. Scores on the APNS scale were used to categorize participants as S, not SPD.
  3. negatively influence performance on the cognitive tasks and should be controlled for in the study. This contradicts our breakdown and what we said in the explanation for answer choice A. Fluid intelligence was positively correlated with verbal fluency in the SPD group. That means we can eliminate this answer choice.
  4. negatively influence scores on the APNS scale and should be controlled for in the study. This is similar to answer choice B. There’s no mention about fluid intelligence negatively influencing scores on the APNS scale. The only thing we know is fluid intelligence positively correlated with verbal fluency in the SPD scale. We can eliminate this answer choice because there’s no evidence in the passage to support it. We’re left with our correct answer, answer choice A.

73) We can approach this question by using our general knowledge to relate verbal fluency and negative priming tasks with Alzheimer’s disease. 

Verbal fluency is retrieving specific information within restricted search parameters. Negative priming is slow, error-prone responses to stimuli that were previously ignored. Alzheimer’s disease is a neurogenerative disease and common cause of dementia. We can see disruptive memory loss, confusion, difficult executing tasks, poor judgment, and personality changes. Biggest takeaway is there’s dramatic neurodegeneration and cognitive dysfunction. Participants should do worse with both tasks. Cognitive dysfunction is going to affect priming, and a verbal fluency test is actually a very common way to screen for dementia.

  1. have no effect on performance on either of the verbal fluency or negative priming tasks. This answer choice contradicts our breakdown because we said performance on both tasks will be adversely affected. 
  2. have no effect on performance on the verbal fluency task and negatively affect performance on the negative priming task. This answer choice also contradicts our breakdown. We said performance on both tasks will be adversely affected, not just the priming task.
  3. negatively affect performance on both the verbal fluency and negative priming tasks. This answer choice is consistent with our breakdown and our definitions. That means we can now eliminate answer choices A and B because both contradicted our breakdown.
  4. negatively affect performance on the verbal fluency task and have no effect on the negative priming task. Same reasoning as A and B here. We’re going to see Alzheimer’s disease negative affect both tasks. We can eliminate answer choice D, so we’re left with our correct answer, answer choice C.

Section Bank: Psychological, Social, and Biological Foundations of Behavior: Passage 10

74) We’ll revisit what the passage says about the stress generation hypothesis and break down what makes it unique in this context. 

Stress generation hypothesis proposes that individuals prone to depression will be exposed to stressful situations because of their behaviors, perceptions, and experiences. Our answer has to be consistent with the idea that negative and stressful life events are influenced by an individual’s characteristics and behaviors. 

  1. more likely to experience independent stressors and less likely to experience dependent stressors. Dependent stressors are events that are due, at least partly, to an individual’s characteristics, feelings, and behaviors. Meaning just like the name suggests, they’re dependent on the individual. Independent stressors are something that happen regardless of your personal behavior, feelings, thoughts, attitudes, and so on. That means proponents of SG hypothesis would expect depression prone individuals to be more likely to experience dependent stressors. We didn’t learn about any explicit differences in independent stressors. What does this mean? We can refine our breakdown. Even though my initial breakdown didn’t mention stressors, we can say these individuals will be more likely to experience dependent stressors.
  2. Answer choice B says more likely to experience independent stressors and just as likely to experience dependent stressors. This answer choice also contradicts our newly refined breakdown of the question. We said individuals prone to depression would be more likely to experience dependent stressors. We don’t expect a change in experiencing independent stressor. Let’s keep comparing.
  3. Answer choice C says just as likely to experience independent stressors and more likely to experience dependent stressors. This answer choice sounds exactly like our new breakdown of the question. Proponents of SG hypothesis would expect depression prone individuals to be more likely to experience dependent stressors. We also didn’t learn about any explicit differences in independent stressors, so it makes sense the depression-prone individuals would be just as likely to experience independent stressors compared to what others expect. We can eliminate answer choices A and B. Both contradicted what we learned in the passage.
  4. Answer choice D says more likely to experience both independent and dependent stressors. This answer choice also contradicts what we learned in the passage. It is true that proponents of SG hypothesis would expect depression prone individuals to be more likely to experience dependent stressors. But it’s not true they would expect these individuals to be more likely to experience independent stressors. That means we’re left with our correct answer, answer choice C: Proponents of the SG hypothesis would expect that, compared to others, an individual prone to depression is just as likely to experience independent stressors and more likely to experience dependent stressors. 

75) With any question that’s worded like this, be careful with the verbiage. 3 of the answer choices listed will be valid criticisms of the use of self-reports, while one will be implausible. Our correct answer is the implausible criticism. The passage mentioned that studies using self-reports found a relationship between childhood maltreatment and developing depression. We’re now focused on the reporting method itself.

There are weaknesses to self-report studies and their validity can be questioned. Patients might misremember the topics covered in the survey. They might exaggerate or under-report symptoms to make their situation seem better or worse. There’s a lot of wiggle room and subjectivity in these studies. 

  1. There is a potential for poor reliability. This answer choice is the textbook reason people would criticize self-reports in this situation. For example, all of the subjects might downplay their childhood maltreatment, but still develop depression in life. This is a valid criticism.
  2. They are vulnerable to subjective interpretation. This was a word we mentioned in our breakdown. Subjectivity means influenced by personal feelings or opinions. That’s exactly what can happen in self-reports. Both A and B are not great options so far. Both are valid criticisms.
  3. The results are not easily compared with those from other measures. This is another valid criticism. This is not exactly a quantitative study, so comparing results can be more difficult. Let’s keep comparing. 
  4. They are labor intensive and expensive to implement. We actually know the opposite to be true. Self-reports can study large samples and variables quickly and easily. The respondents are essentially answering questions about their experiences and beliefs, so they’re not labor intensive or expensive. This is not a valid criticism, meaning it’s actually our best answer in this case. We can eliminate answer choices A-C for all being valid criticisms and, therefore, not answering the specific question being asked. We’re left with our correct answer, answer choice D: They are labor intensive and expensive to implement.

76) To approach this question, we want an answer choice that considers the idea that long-term exposure to stressful events can complicate depression-related matters. And that’s specifically in individuals with vulnerabilities to depression and individuals that had previous episodes of depression.  

  1. General adaptation syndrome. General Adaptation Syndrome, which is also known as GAS, is a theory that demonstrates that the threat of stress can be debilitating if it’s continuous. That sounds exactly like what we mentioned in our breakdown.
  2. Fight-or-flight response. Fight-or-flight response is a physiological reaction that occurs in response to a perceived harmful event, attack, or threat to survival. It’s not so much related to long-term exposure to stress. Answer choice A remains superior. 
  3. Activation-synthesis model. This answer choice has more to do with dreaming and sleep. Less to do with long-term exposure to stress. Answer choice A remains superior. 
  4. Answer choice D says Long-term potentiation. Long-term potentiation has to do with strengthening of synapses based on recent activity. Even though it has the words “long-term” in the name, this focuses more on synapses, and not on stress. It’s not often AAMC throws a distractor like this in their options, but just be careful to not pick an unrelated option just because it sounds like something in the question stem. We’re sticking with the best answer choice, answer choice A.

77) I actually defined dependent stressors in question 76. I said dependent stressors are events that are due, at least partly, to an individual’s characteristics, feelings, and behaviors. It’s almost cyclical. The stressors are dependent on the individual. Independent stressors are something that happen regardless of your personal behavior, feelings, thoughts, attitudes, and so on. That means we want to explain some sort of behavior or characteristic that ultimately causes stress for the person. It’s partly or fully dependent on the person doing something. We want a dependent stressor associated with reassurance.

  1. The need for reassurance. The need for reassurance itself isn’t a characteristic or behavior. And we ultimately want something that’s caused or driven by a person’s behavior or characteristics. This isn’t an action. Key point is we want something that actually causes the need for reassurance. Not the need itself. 
  2. Interpersonal tension caused by the need for reassurance. This answer choice suggests the person acts in a specific behavior. That behavior is almost a choice in this situation. Just like we said in answer choice A, the need for reassurance isn’t an action or behavior. This question essentially saying the dependent stressor “interpersonal tension” is due to the behavior or characteristic “need for reassurance”. That’s not something a person does. This answer choice is iffy at best. Let’s try and find a better answer/
  3. The isolation resulting from others being driven away. This answer choice suggests the person drives others away, and that causes isolation. Driving people away can be a behavior, or action in this case. By driving others away, that causes the dependent stressor “isolation”. I feel like this could be interpreted differently also. We might say others being driven away isn’t a choice or behavior, but this is the best option so far, just because driving others away can actually be an individual’s behavior.
  4. The depression that results from the isolation. This answer choice suggests the individual isolates, and that causes the dependent stressor: depression. While isolation can be something within a person’s control, it’s not necessarily as much of a behavior as driving other people away. Isolation isn’t as much of an action as actually driving people away. That means we can still stick with our best answer, answer choice C: The isolation resulting from others being driven away.

78) To answer this question we’ll go through some of the triggers or causes of depression, but within the context of the SG hypothesis and the passage. 

We have the paragraph from the passage that focuses on the stress generation hypothesis here. Right away, we’re told this model accounts for chronic depression which we said means it’s recurring. Exhibiting behavior that leads to stressful events, long-term exposure to stress, and a preexisting vulnerability to depression all sound like something the therapist would want to look into. Looking at the passage, it says the stress generation hypothesis proposes that individuals who are prone to depression are more likely to exhibit behaviors and cognitions that lead to stressful situations. That also ties into individuals with preexisting vulnerabilities. We also know from earlier in the passage, there’s a high rate of recurrence in people who have had prior episodes. And lastly, there could be a focus on dependent stressors. We’ve said dependent stressors are events that are due to an individual’s characteristics, feelings, and behaviors. A depressive episode would inevitably influence an individual’s feelings and behaviors and could lead to additional depression and episodes. We can use all of this information and we can try and pick an answer choice that’s consistent with these triggers or causes.

  1. lowering the severity of initial depressive symptoms. This answer choice does not get brought up much in our breakdown. In fact, if there are initial depressive symptoms, that’s more likely to lead to additional episodes. Lowering the severity would be helpful in general, but within the context of this specific question, that likely wouldn’t be a focus.
  2. preventing the recurrence of depressive episodes. This answer choice is one of the things we brought up in our breakdown. We said by preventing episodes of depression in the first place, we can reduce the number of recurring episodes. What was one of the reasons? Because dependent stressors are due to an individual’s feelings and behaviors. By cutting down on depressive episodes and behavior, the individual is theoretically reducing the risk, and breaking the cycle. We like this answer better than answer choice A.
  3. preventing stressful life events. This would be great in a vacuum, but is this practical? The stress generation hypothesis focuses on dependent stressors, but that doesn’t mean we can also prevent independent stressors. Unfortunately, those can still happen. Imagine an earthquake, or an illness, or a recession. We want to reduce the effect of these stressful events, but we can’t prevent them entirely.
  4. understanding the unique stress cycle of each individual. While this might help on a more micro level, the therapist is more concerned with a widely applicable treatment. We can do an example of a careless driver that crashes their car. Obviously, a ton of negative comes out of this situation, and the situation happened because of the behavior of the individual. Understanding that unique stress cycle can be helpful for that specific person. But that might not be as helpful for someone that’s not a careless driver, or in that same situation. We can also eliminate answer choice D. We’re going to stick with the answer choice that’s the most broad and practical here. Answer choice B: The therapist will pay a great deal of attention to preventing the recurrence of depressive episodes.

79) To answer this question, we want to link depression and stress, and specifically describe the direction of the relationship. 

First of all, the SG hypothesis proposes that individuals who are prone to depression are more likely to exhibit behaviors and cognitions that lead to stressful situations. What does that sound like to us? There’s a focus on dependent stressors. Those are events that are due to an individual’s characteristics, feelings, and behaviors. It’s almost cyclical. An individual has a certain behavior, that behavior leads to depression, and that depression causes the individual to continue to act a certain way. The cycle continues.

On the other hand, independent stressors are something that happen regardless of your personal behavior, feelings, thoughts, and actions. Think of a driver driving recklessly because they got dumped by their significant other. They aren’t paying a ton of attention to the road and they rear-end a car parked in a driveway. The owners of both cars inevitably have a negative outcome here. But what’s the difference? The careless driver was sad, that influenced their behavior, and they acted in a way that lead to a negative outcome. The owner of the parked car is dealing with an independent stressor. That individual parked their car in their driveway and didn’t have anything to do with the event. The driver’s behavior is what caused the collision here. We want an answer choice that explains that difference.

  1. Both independent and dependent stressors are unidirectionally related to depression. This answer choice is inconsistent with our breakdown. Independent stressors are unidirectionally related to depression, think back to our car example. The owner of the parked car experienced a negative event. But their behavior and characteristics didn’t cause that negative event. On the other hand, the careless driver experienced a breakup, that caused them to drive recklessly, the poor driving led to another negative event: a car accident. Dependent stressors are more cyclical, independent is more unidirectional. 
  2. Independent stressors are unidirectionally related to depression, and dependent stressors are reciprocally related to depression. This is exactly what we said in our breakdown. Independent stressors are unidirectionally related to depression. Depression is not what influences behavior and leads to additional depressive events.
    On the other hand, dependent stressors are more cyclical: they’re events that are due to an individual’s characteristics, feelings, and behaviors. They’re related reciprocally-there’s mutual action. That means we like answer choice B.
  3. Dependent stressors are unidirectionally related to depression, and independent stressors are reciprocally related to depression. Both parts of this answer choice go against our breakdown and the reasoning we used for answer choices A and B. That means we can eliminate answer choice C.
  4. Both independent and dependent stressors are reciprocally related to depression. This is similar to answer choice A. Dependent stressors are reciprocally related to depression, but we said independent stressors are unidirectionally related. This answer choice is only half right, which unfortunately also means it’s half wrong. It contradicts what we said in the breakdown. That means we’re left with our correct answer, answer choice B: Independent stressors are unidirectionally related to depression, and dependent stressors are reciprocally related to depression.

Section Bank: Psychological, Social, and Biological Foundations of Behavior: Questions 80-84

80) Visual processing is the interpretation of otherwise raw sensory data to produce visual perception. If you are not comfortable with visual processing, you want to make sure to review AAMC’s content outline on our website: https://jackwestin.com/resources/mcat-content/vision/visual-processing. We’re going to think about visual processing, but we have to consider parallel processing specifically. Parallel processing is the use of multiple pathways to convey information about the same stimulus. It starts at the level of the bipolar and ganglion cells in the eye, allowing information from different areas of the visual field to be processed in parallel.

  1. spatial information. Spatial information can be processed without conscious effort. Think about sitting in a quiet classroom before class. Often you might sit in a seat and not even think about most of your surroundings. You will notice the space around you, but you don’t have to make a conscious effort to do so. It’s expected and normal. 
  2. temporal information. Temporal information can be processed without conscious effort. Similar to the example in answer A, think about getting to a class 10 minutes early. You can sit in your chair and relax for 10 minutes. You know time is passing without having to make a conscious effort to do so.  
  3. the frequency of specific events. As you’re sitting in a classroom before class, you’ll notice students shuffle in. You might not know exactly how many students come in, but you don’t consciously think about every student that comes through the door. If the professor asks you how many students are in the class, you can probably make an educated guess (even if you don’t get it exactly or particularly close, you wouldn’t guess 5 people in a 300 person lecture).
  4. novel information. Novel information is new, and you have to be consciously aware of it. Think back to our example of sitting in a quiet classroom before class starts. There can be conversations going on around the lecture hall that you don’t pay attention to. It’s all expected and something you’re used to. However, imagine someone comes in with speakers and starts playing loud music. That an unexpected, new stimulus. It requires conscious effort to process this new information. This is going to be our best answer.  

81) We have to different rates being reported in this situation. There’s a high rate of one variable (insomnia) and that correlates to a low rate of the other variable (detecting sounds of birds chirping). That’s a negative correlation because as one variable changes, the other variable moves in the opposite direction. 

  1. Positive, signal detection. First part of this answer choice goes against our breakdown. We want an answer that talks about negative correlation. What about the rest of our answer? At its most basic, signal detection theory states that the detection of a stimulus depends on both the intensity of the stimulus and the physical/psychological state of the individual. Basically, we notice things based on how strong they are and on how much we’re paying attention. This is consistent with the rates being tested in the question stem.
  2. Negative, signal detection. First part of this answer choice is consistent with our breakdown. Furthermore, we mentioned signal detection is going to be the correct phenomenon described.
  3. Positive, difference threshold. Positive is the opposite of what we expect. The minimum amount of change in sensory stimulation needed to recognize that a change has occurred is known as the difference threshold. We don’t like either part of this answer choice.
  4. Negative, difference threshold. While we like the “negative” correlation type, I mentioned we prefer signal detection to difference threshold in the second part of each answer choice. We can stick with answer choice B as our best answer.

82) This is a common phenomenon that a lot of people will report, and something to keep in mind in the future! There’s a level of “mind over medicine” where patients will report treatment works better when they mentally associate positives with the experience and the treatment. In the question stem, we’re comparing a positive experience with the usual doctor, with an experience that might not have been as positive in the new location. We have to explain the reason for this difference.

  1. The morphine interacted with another medicine the patient was taking. There’s no way to know if there is another medicine the patient is now taking that they were not taking before. We would need more information in the question stem to consider this answer choice.
  2. The morphine was given in a new environment. This is exactly what is happening. The patient is not in their positive, comfortable environment. As a result, we don’t have the same effectiveness of the dose. Unlike answer choice A, we are explicitly told location changes and the doctor changes. This question and this answer come down to using the information in the question stem and not making any broad assumptions that the test-maker does not want us to make.
  3. The doctor’s office used a different drug manufacturer. This is going to be similar to answer choice A. We don’t know for sure if this can be true or not. We would need more information in the question stem to consider this answer choice.
  4. The patient’s body became sensitized to the same dose of morphine. This is going to be similar to answer choice A. We don’t know for sure if this can be true or not. We would need more information in the question stem to consider this answer choice. 

83) The human brain is composed of a right and a left hemisphere, and each participates in different aspects of brain function. A longitudinal fissure separates the human brain into two distinct cerebral hemispheres connected by the corpus callosum. The corpus collosum is a wide, flat bundle of neural fibers beneath the cortex that connects the left and right cerebral hemispheres and facilitates interhemispheric communication. In this question, that corpus callosum is severed. This is a very broad question, but the only information we know besides the corpus callosum being severed, is the person says “ball” when asked what they see. One thing I do want to note. This question presents an all-or-nothing type of approach with which part/side of the brain is responsible for which actions. In reality, these processes and how everything is broken up is much more complex. For the sake of this question and the MCAT, we will keep it simple. 

  1. The letter b is projected to the left visual field, followed almost immediately by “all,” at the same location. Each eye is going to project contralaterally; the left visual field projects to the right side of the brain, while the right field projects to the left side of the brain. We expect the language center of the brain (in the left side of the brain) to process what is projected to the right visual field.
  2. The word “ball” is projected to the right visual field, while “room” is presented to the left visual field. The left visual field projects to the right side of the brain, while the right field projects to the left side of the brain. Language center of the brain processes what is seen, so the word “ball” would be processed by the language center (in the left side of the brain), while “room” would not. 
  3. The word “base” is projected to the right visual field, while “ball” is projected to the left visual field. Reasoning here is going to be the exact opposite of what we came up with in answer choice B. We would expect the person to say “base” given what we know and what we said in our breakdown.
  4. The word “basketball” is projected to the left visual field. The entire word being projected to the left visual field would not cause the person to break it up into two words and only repeat the right side of the word. We want the word “ball” projected to the right visual field like we see in answer choice B. We can stick with answer choice B as our best option.

84) In psychology, “cognitive dissonance” describes the mental stress or discomfort experienced by an individual who holds two or more contradictory beliefs, ideas, or values at the same time, or is confronted by new information that conflicts with existing beliefs, ideas, or values. When we experience cognitive dissonance, we are motivated to decrease it because it is psychologically, physically, and mentally uncomfortable. We can reduce cognitive dissonance by bringing our cognitions, attitudes, and behaviors in line—that is, making them harmonious. We can change the discrepant behavior, change our cognitions through rationalization or denial, or add a new cognition. 

  1. adapt their attitudes to be more in line with the council member. While the two friends may not like to hear it, this is going to be the most likely. We reduce dissonance by bringing our cognitions, attitude, and behaviors in line. We expect they will change their behaviors and beliefs to match their behavior while working for the city council member.
  2. quit the internships after a period of time. It’s possible this happens, but it’s more likely the friends will change their attitude and beliefs to be more in line with those of the council member. 
  3. continue in the internships but retain their original beliefs. This seems very unlikely. While the friends may try to hold on to their beliefs, it’s more likely they will change their beliefs as they continue to work for the city council member. We would need more information about the specific beliefs to make this claim.
  4. do as little work as possible so as not to advance the council member’s agenda. This is a change in behavior, but it’s easier for the friends to change their beliefs rather than risk being fired for doing very little work. Changing their beliefs would be the best way to decrease cognitive dissonance. Answer choice A is our best answer. 

Section Bank: Psychological, Social, and Biological Foundations of Behavior: Passage 11

85) In other words: Children aged 3-6 are in which stage of Piaget’s model of cognitive development? This answer relies exclusively on our general knowledge outside of finding the ages of the children in the passage. We need to classify this age range within Piaget’s stages of cognitive development.

  1. Sensorimotor stage. This corresponds to ages 0-2 years old, sometimes 0-1. It involves coordinate of senses with motor responses and we see object permanence. This is too young for what we’re looking for. 
  2. Preoperational stage. This corresponds to ages 2-7. Children start to talk and use symbolic thinking. This is exactly the range we’re looking for. This answer is superior to option A.
  3. Formal operational stage. This corresponds to an age range of 11 years and older. This is when we see theoretical and hypothetical thinking. There’s abstract thoughts and reasoning, strategy, and planning. But this is older than what we’re looking for in this situation. 
  4. Concrete operational stage. This corresponds to ages 7-11 years old. Like the name suggests, there’s now concrete thoughts and logical reasoning. This is slightly older than the target age group in the passage. We’re left with our correct answer, answer choice B: Preoperational stage

86) Recall from the passage there was the rating system to measure aggression, but how else could we achieve a similar result? We’ve already been given the background on the aggression scale and its purpose. We’re told the aggression scale allowed the researchers to isolate socially-influenced aggressive behaviors. Now we want another method that shows whether aggression is being modeled, as opposed to already learned.

  1. The observations of teachers unfamiliar with the children. This answer choice defeats the purpose of the observational ratings from Study 1. The only reason the teachers were asked to rate the students is because the teachers already knew the aggressive tendencies of the children. These new teachers wouldn’t know if any aggressive behavior is modeled or previously learned. Let’s try and get a better answer choice.
  2. A different rating scale including relational aggression and obedience. Doing a different rating scale that allows obedience would not be relevant to aggression and how it manifests. This answer choice is out of scope.
  3. Answer choice C says A biographical sketch of the children’s social life up until the study. A biographical sketch would provide information about the child’s background and past. This allows for the researchers to see if there was past, aggressive behavior or not. This is a good answer, it gets us the information we needed to conduct Study 1 properly. This is superior to answer choices A and B.
  4. Answer choice D says A heritability ratio for aggression as determined by behavioral genetics. The study wasn’t focusing on genetics. It was instead focused on modeled aggressive behavior versus aggressive behavior learned previously. This answer choice is out of scope. We’re left with our correct answer, answer choice C.

87) To answer this question, we can revisit Study 2, and specifically the findings.

We have the 2nd paragraph from our passage above. A few things I want to note: We were testing whether film violence is in instigator of violence, or it relieves individuals of hostile impulses. The last sentence says Results indicated that subjects who were given the stigmatizing example applied a higher number of shocks than subjects who were given the sympathetic portrayal. Seeing violence portrayed in a film didn’t make the subjects inherently more apt to behave aggressively or to be calmer. That means the influence of the violence came down to context.

  1. Media has little effect on the development or learning of aggression. We said the influence of the violence in media comes down to context. There was certainly an effect on aggression. We saw the two groups respond differently to the violence in the film. This contradicts what we read in the passage.
  2. Aggression may be lessened via the influence of the media. There’s no way to make this connection based on what we saw in the passage. We only compared two groups that saw aggression. One group was less aggressive than the second group, but we don’t have a baseline. We don’t know if both groups were made more aggressive, we just know one group was comparatively more aggressive in the end.
  3. Aggression is fostered via the influence of the media. This is similar to answer choice B. We only compared two groups that saw violence, but we didn’t have a baseline. We don’t know how aggressive either group would’ve been had they not seen the video. There’s no way we can say aggression is lessened or fostered. 
  4. The impact of media on aggression is situationally dependent. This answer choice is right on the money. We said it all comes down to context, and that’s exactly what this answer choice is saying. Depending on the situation, subjects were either more aggressive or less aggressive compared to the other group following exposure to media. Best way to put it is the impact is situationally dependent. We’re left with our correct answer, answer choice D. 

88) Considering this is a behavioral passage, we’re likely filling in that blank with a vocabulary word. This answer is twofold: We’ll have to relate the researchers in Study 2 to media function. then we’ll have to use our general knowledge to likely put a vocabulary term to the function.

Here we have the second paragraph from the passage, and we’re focused on the hypotheses. First hypothesis says violent films were cathartic. Second hypothesis suggests film violence as an instigator of violence. These researchers are both suggesting media can influence how we act. One group believes media can relieve us of hostile impulses, while the other group believes media can incite violence from us. Both suggest media can shape an individual’s behavior to an extent. 

  1. the looking-glass self. This suggests people see themselves based on how they believe others perceive them during social interactions. We imagine how we appear to others, we imagine judgment related to that appearance, and we develop ourselves based on this judgment. There’re not any judgments coming from the media. This is out of scope.
  2. the generalized other. This is the common behavioral expectations of society. It informs us about how our behavior is shaped by our surrounding community and their collective expectations. The media isn’t functioning as the generalized other. This is also out of scope.
  3. social capital. This is the advantage one gains through their social network. That’s not relevant in this situation. The test maker just added this answer choice as a distractor, just because it’s a term you’ll see in the behavioral section. 
  4. an agent of socialization. Just knowing the definition, media is an example of an agent of socialization, and especially in Study 2. Agents of socialization can impress social norms upon an individual, so they can shape our social values, behavior, or beliefs. That’s exactly what’s happening in Study 2. Violence in media is influencing aggressive behavior in the people consuming it. We like this answer choice better than the three previous options. We’re left with our correct answer, answer choice D: an agent of socialization.

89) There was no denying the fact that was a different proportion of shocks that came from the subjects in the two groups. We know there’s evidence that both the film clip and the judgment-based introduction affected the proportion of shocks. There are an infinite number of ways Study 2 could be modified, but we want something that adds to this evidence. We want an answer choice that shows the film clip and the introduction of the protagonist both influenced the number of shocks.

  1. Question each subject beforehand relative to their involvement in contact sports. This answer choice is out of scope. Contact sports have nothing to do with aggressive behavior within the context of the study or the passage. This is added as a distractor. 
  2. Use violent and nonviolent clips, with positive and negative protagonist introductions. This answer choice would add evidence suggesting both the clip and the introduction to the protagonist were the reason for the different proportion of shocks. By controlling for one variable, they can change the other variable and track the different proportions of shocks. We like this answer choice better than answer choice A.
  3. Show a boxing clip with an indeterminate outcome between opponents. An indeterminate outcome means we don’t know who wins. Meaning the positive protagonist could win or lose, or the negative protagonist could win or lose. It’s likely the subjects wouldn’t have as strong of feelings if there’s an indeterminate outcome. 
  4. Allow subjects to choose whether they want to shock or punch the compatriot. This is an amusing visual, but we don’t know the effect of the shock, or how hard the subjects can punch. That means we don’t know if one option is worse than the other, or if all of the subjects would treat one worse than the other. Too many unknowns in this situation. We’ll stick to our best answer, answer choice B: use nonviolent and nonviolent clips, with positive and negative protagonist introductions. 

90) We can approach this question by recalling the findings in Study 2, specifically as they relate to the stigmatizing example. We will then use our general knowledge to define symbolic interactionism. We’ll find the answer choice that’s most consistent with a symbolic interactionist’s interpretation.

We have the 2nd paragraph from our passage. The last sentence is where we got our results. It says Results indicated that subjects who were given the stigmatizing example applied a higher number of shocks than subjects who were given the sympathetic portrayal. We’re interpreting this from the lens of a symbolic interactionist. The moment you see symbolic interactionism, think interpretation. Symbolic interactionism focuses on interactions and relationships among individuals in a society. Our experiences and interactions add subjective meaning to things. We interpret things based on these interactions. Furthermore, people will change based on their interactions with objects, events, ideas, and other people. 

In Study 2, the individuals interpreted the scoundrel as a negative figure. After viewing the boxing scene in which that stigmatizing protagonist is beaten, the subjects responded by applying more shocks. Let’s see which of the 4 answers choices is consistent with the symbolic interactionist’s interpretation.

  1. created a negative label the participants interpreted and responded to in their behavior. This is consistent with a symbolic interactionists’ interpretation. It’s also consistent with our breakdown. The scoundrel example created a negative label. The subjects labeled the scoundrel as a bad guy. The subjects all interpreted and responded. How do we know this? Because their poor compatriots were getting shocked like there’s no tomorrow! I’m liking this answer choice.
  2. operated as an effective sorting mechanism for administering shocks. This has more to do with the mechanism of the experiment itself, not so much an interpretation of the findings. The symbolic interactionist is all about interpreting things and the meaning we add to things, that’s the key to this question. Answer choice A is superior to answer choice B
  3. inequitably predisposed certain participants toward receiving additional shocks. This may or may not be true, but this isn’t related to the findings in Study 2 themselves. Sure, the symbolic interactionist might feel some guilt for the poor participant getting shocked, but that’s not how they would interpret the findings. We’re focused in interpreting things and adding meaning to things. Answer choice A still remains superior.
  4. supports a rational basis for differentially administering shocks to participants. This again has more to do with the mechanism of the experiment itself, and not an interpretation of the findings. The subjects in the study interpreted the scoundrel as a negative person. That led to increased shocks for the compatriots. We’re sticking with our best option here, answer choice A. It’s the only one that talks about interpretation and interaction. Those are key things to look out for when dealing with symbolic interactionism.

Section Bank: Psychological, Social, and Biological Foundations of Behavior: Passage 12

91) This answer is going to come from recalling the main parts of the 2012 study then applying our general knowledge to classify the data from the study. It’s a fairly open-ended question so we won’t get too specific just yet, but we can quickly pull up an excerpt from the passage.

We have the 2nd paragraph from the passage here that goes over the 2012 study. What are we looking for specifically here? The data! This question has to do with the data. About 2/3 of the way down in the excerpt, it says The students completed an assessment after completing a course of physician-patient communications. That means the researchers are getting data by the students completing an assessment following this course. 

Next sentence says Findings from the study showed that students with a mastery goal were more likely to rate themselves as being able to handle patient interactions well, and were less likely to report maladaptive coping with stressful situations. There’s quite a bit to unpack here if you didn’t do so during the passage:

The study is applied to a student’s sense of confidence regarding patient interactions and tolerating frustration. In general, if you’re more confident, you’ll tend to perform better. Why would you be more confident? If you’ve taken the time and put in the work to enhance your knowledge. You’re not focused on extrinsic factors or factors you can’t control. By focusing on a mastery goal, these students are aiming to enhance their own knowledge, and will therefore be readier to handle patient interactions properly. Additionally, we’re told these students are less likely to report maladaptive coping with stressful situations. Maladaptive coping can also be considered negative coping. So that can mean self-harm, substance abuse, binge eating, etc: Multiple negative ways to try and cope with stress and anxiety. There’s no doubt the students focused on a mastery goal had to deal with stressful situations. Again, that’s the nature of the profession. But the way these students coped was more positive, and less destructive. The mastery goal ended up improving confidence, and the ways in which these students coped with stress. Let’s keep this in mind and see which answer choice most likely describes this part of the passage.

  1. longitudinal. Longitudinal research describes a research study that would follow the medical students in the study at multiple time points. This contradicts the passage and what we just looked at in our breakdown. This assessment was done at a set point in time, not at multiple time points. 
  2. cross-sectional. Cross-sectional research describes study done at a specific point in time. That’s exactly what’s happening with this study of medical students. They were given an assessment at a set point in time. We like this answer better than answer choice A. 
  3. ethnographic. Ethnographic research: research that studies people or cultures in their own environment. This is not what was being studied, so we can eliminate answer choice C. It contradicts the study description in the passage. 
  4. experimental. Experimental research is research in which scientists manipulate variables to test the difference between an experimental and control group. Again, not what’s taking place here. We can eliminate answer choice D for contradicting the passage. We’re left with answer choice B: cross-sectional data. 

92) Let’s begin by defining incentive theory. Within the context of the passage, incentive theory says our behavior is dictated by a desire for external rewards. These external rewards, or incentives, provide motivation. That anticipated reward is what motivates and encourages us. Behavior is extrinsically motivated, and people are more motivated if they receive a reward after doing something. We want an answer choice that closely aligns with this.

  1. Conflict theory. Conflict theory says that society is in a state of perpetual conflict because of competition for limited resources. So not exactly the same as incentive theory, but people are still looking for power and resources. Remember, we’re looking for the theoretical framework that’s most closely aligned with incentive theory. That means we’re still holding on to answer choice A.
  2. Exchange theory. Exchange theory assumes people will try and maximize their benefits and rewards while minimizing costs and punishments in any social relationship. First part certainly sounds like it’s aligned with incentive theory. Bigger incentives mean bigger benefit. We can keep this answer choice, and it’s a superior answer to answer choice A which was out of scope. 
  3. Structure functionalism. Structure functionalism says society is made of parts working together to serve the needs of its individuals. Everything has a certain function. Again, this is out of scope and not the most aligned with incentive theory. We can eliminate answer choice C. 
  4. Symbolic interaction. Symbolic interactionism functions to explain how individuals act in society. Using this smaller focus, it can be expanded to look at the interactions of larger social groups to explain social change. Again, out of scope in this case. We can eliminate answer choice D and we’re left with our correct answer, answer choice B: Exchange theory.

93) The key to the question is we’re focusing on the findings on the 2012 study. The author mentions medical students reporting coping with stressful situations, so we can revisit the passage.

The author says about the 2012 study, Findings from the study showed that students with a mastery goal were more likely to rate themselves as being able to handle patient interactions well, and were less likely to report maladaptive coping with stressful situations. This is the only time in the passage there’s any mention of coping. These students are less likely to report maladaptive coping, or negative coping, with stressful situations. Maladaptive coping can mean self-harm, substance abuse, binge eating, and other negative ways to try and cope with stress and anxiety. If the students aren’t engaging in maladaptive coping strategies, we expect them to engage in more adaptive coping strategies.

  1. Status-seeking. Status-seeking involves compensating by trying to impress others and seek attention and achievement. This is a maladaptive coping strategy. 
  2. Aggression. Aggression can be shown through different forms of abuse, blaming, criticizing, or any sorts of negative verbal and physical attacking. Another maladaptive coping strategy. 
  3. Dependence. Dependence can mean relying too much on others, being passive or submissive. This can involve clinging or people-pleasing. Again, maladaptive coping strategy. 
  4. Support-seeking. Support-seeking involves coping through using emotional support through friends, family, colleagues, or the community in general. This is an adaptive coping strategy. We can eliminate answer choices A-C because they were maladaptive coping strategies. We’re left with our correct answer, answer choice D: Support-seeking. 

94) We want to note the question stem explicitly says “as described in the passage.” The author mentions medical students engage in learning activities to pursue goals. One goal mentioned is performance-approach. The author says this means to receive rewards or recognition. These rewards and recognition aren’t given out for no reason! Students will work toward that goal by putting in time and effort. Being rewarded and recognized for their hard work is a big motivating factor for students. They’ll look to perform better, and even outperform their colleagues and classmates. We can use this definition and breakdown to see which of our answer choices fits best.

  1. can be conceptualized as formal or informal sanctions. We’re looking at social norms here. Sanctions can be positive or negative (so rewards or punishment), but they’re a form of external control. This is consistent with the external motivation we might get from rewards and recognition. These can be formal or informal; I am liking this answer choice.
  2. are often the result of latent functions. Latent functions are unintended and unrecognized outcomes. This is not consistent with performance-approach outcomes because these students are intending for certain outcomes. 
  3. can be associated with role exit in social interactions. Role exit is disengaging from a role to take up a new one. The initial role is part of the person’s identity, but they ultimately exit that role. Role exit in social interactions is usually not consistent with rewards or recognition, although it can be. We can still stick with our superior answer, answer choice A. 
  4. are direct manifestations of the individual’s ascribed status. Ascribed status is involuntary: a person might be assigned a certain status at birth. That’s not consistent with working toward a goal and being rewarded and recognized. We’re left with our best answer, answer choice A: Performance-approach outcomes can be conceptualized as formal or informal sanctions

95) To answer this question, we’ll rely on our behavioral vocabulary. 

Reinforcers can be primary, meaning linked unconditionally to a behavior. Primary reinforcers occur naturally and don’t need to be learned. Typically, they’ll satisfy basic survival needs.

Secondary reinforcers require deliberate or conditioned linkage to a specific behavior. A stimulus reinforces a behavior after it’s been associated with a primary reinforcer. The classic example is money. You can satisfy basic survival needs using money. We’re focused on identifying the answer choice that’s a secondary reinforcer.

  1. water. This is a primary reinforcer. Remember the general rule I said was primary reinforcers will satisfy basic survival needs. We’re looking for a secondary reinforcer that reinforces, or is associated with, a primary reinforcer. 
  2. sexual activity. This is another primary reinforcer
  3. money. This was actually the exact secondary reinforcer I said in the breakdown. This is the main example people give for secondary reinforcers. Money can be used to buy goods and services that are essential for basic survival. We can eliminate answer choices A and B because both contradicted our definition of secondary reinforcer.
  4. food. Food is another primary reinforcer. That means we’re left with our correct answer, answer choice C: money.

96) Let’s start by defining some terms. Self-efficacy is believing in your own competence and effectiveness. Confidence is a firm trust and reliance in something or someone. We’ve all felt these in certain situations. You guys will certainly feel these shortly in your careers. In this case, we can visualize both of these. In a medical student, when would self-efficacy and confidence in patient interactions be seen? The best way to approach this question, is to see the situations presented in our answer choices.

  1. The Thomas theorem. Thomas theorem says “if men define situations as real, they are real in their consequences.” Essentially, the outcome depends on the way a situation is interpreted, not the actual situation itself. There’s subjectivity in our realities. This is outside the scope of what’s being asked here.
  2. Impression management. Impression management is the process of consciously making behavioral choices in order to create a specific impression in the minds of others. In this case, we’re talking about in the minds of patients. You act with a certain confidence and high self-efficacy in front of patients. You want to present yourself as confident to gain the patient’s trust and show you’re able to take care of them. This is certainly a time when it’s likely to manifest. This answer choice is superior to answer choice A.
  3. Back stage self. Backstage self is when players are together, but no audience is present. This can mean behavior that’s unacceptable when performed in front of the audience. You guys will (or maybe already have) experience this in your careers. I advise you to always be professional and confident, but the vibe when you’re dealing with patients versus when you’re with your colleagues can be totally different. I’m sure you guys experience this in all kinds of situations in your life already. But in this case, the back stage self would not be when interacting with patients, and that would not be when you expect the most confidence and self-efficacy. 
  4. Hawthorne effect. This would involve the students changing their behavior because they’re aware of being observed. This usually applies more to pretending to be working harder or longer in a professional setting. Not so much actually interacting with patients. The definition contradicts what we’re looking for in the question stem. We’re left with our correct answer, answer choice B: Impression management.

Section Bank: Psychological, Social, and Biological Foundations of Behavior: Questions 97-100

97) Gender is defined as a social construct. Gender is the division of groups of people by associate roles, expectations, and stereotypes in a culture that is shaped by religious, political, legal, philosophical, linguistic, and other traditions. “male” and “female” are considered sex assignments, while “man” and “woman” are considered genders and gender is a spectrum. 

A social constructionist view of gender looks beyond categories and examines the intersections of multiple identities and the blurring of the boundaries between essentialist categories. This is especially true with regards to categories of man and woman or masculine and feminine, which are viewed typically as binary and opposite. Gender is internalized and acquires significance for the individual. We are aware that others evaluate and characterize our behavior on the parameter of gender. Social constructionists would say that gender is interactional rather than individual; it is developed through social interactions.

  1. biologically determined and immutable. This goes against our breakdown of the question. Immutable means it cannot be changed, but a constructionist would argue gender is fluid.
  2. binary and the product of ingrained socialization processes. Gender is developed through social interactions, but is not simply binary. Gender is fluid.
  3. fluid and subject to social processes of meaning-making. As I mentioned, gender is developed through social interactions. Gender is a spectrum, fluid, and is subject to social processes of meaning-making. We like this answer choice as it is consistent with our breakdown and what a constructionist would say. 
  4. binary, yet interchangeable depending on context. A constructionist would not agree with this statement. This is going to be similar to answer choice B. We can say gender is fluid and subject to social processes of meaning-making. Answer choice C is going to be our best answer.

98) George Herbert Mead studied the self, a person’s distinct identity that is developed through social interaction. In order to engage in this process of “self,” an individual has to be able to view him or herself through the eyes of others. That’s not an ability that we are born with. Through socialization we learn to put ourselves in someone else’s shoes and look at the world through their perspective. This assists us in becoming self-aware, as we look at ourselves from the perspective of the “other.”

Mead believed that there is a specific path of development that all people go through. During the preparatory stage, children are only capable of imitation: they have no ability to imagine how others see things. They copy the actions of people with whom they regularly interact, such as their mothers and fathers. This is followed by the play stage, during which children begin to take on the role that one other person might have, or role taking. During the game stage, children learn to consider several roles at the same time and how those roles interact with each other. They learn to understand interactions involving different people with a variety of purposes. 

Finally, children develop, understand, and learn the idea of the generalized other, the common behavioral expectations of general society. By this stage of development, an individual is able to imagine how he or she is viewed by one or many others—and thus, from a sociological perspective, to have a “self.”

The Me component of the self is the more socialized self, meaning they act according to social norms. We want an answer choice consistent with this.

  1. An individual studying for an exam instead of going to a party. This is exactly what we’re looking for. This individual knows that studying for the exam is necessary to perform well on the exam. A good score on the exam will, theoretically, lead to more success in life. This is an expression of the Me component of the self.
  2. An individual skipping work because they want to go to a concert. This is the opposite of what we want here. The individual is skipping work and breaking social norms here. That’s not what we’re looking for in this question, this is more an expression of the I component of the Self.
  3. An individual ignoring the emotional needs of a significant other. Reasoning here is going to be the same as answer choice B. This goes against social norms. Answer choice A remains superior.
  4. An individual dressing inappropriately for a job interview. Reasoning here is going to be the same as answer choice B. This goes against social norms. Answer choice A remains the best answer choice.

99) Social mobility typically refers to vertical mobility, which is the movement of individuals or groups up or down from one socioeconomic level to another, often by changing jobs or through marriage. In some instances, social mobility is used to refer to horizontal mobility, which is the movement from one position to another within the same social level, as when someone changes between two equally prestigious occupations. In the question stem, the former is happening as the individual experiences vertical mobility. 

  1. Intragenerational. Intragenerational mobility refers to a change in social status over a single lifetime. This is not what happened to the individual in this case. 
  2. Intergenerational. Intergenerational mobility refers to the phenomenon whereby a child attains higher or lower status than their parents. This is out of scope here. We don’t know about the individual’s family.
  3. Horizontal. We mentioned horizontal mobility is the movement from one position to another within the same social level. 
  4. Vertical. Vertical mobility is the movement of individuals or groups up or down from one socioeconomic level to another, often by changing job. That’s exactly what’s happening with the individual in the question stem. We can pick answer choice D as our correct answer.

100) McDonaldization is a phenomenon that occurs when society, its institutions, and its organizations are adapted to have the same characteristics that are found in fast-food chains. These include efficiency (optimal method for accomplishing a task and minimizing time), calculability (quantifiable objectives-high quantity equals quality), predictability and standardization (same service at any McDonald’s chain), and control (employees are standardized and use technology). Make sure to pay close attention to the question stem. We want a scenario that is NOT a representation of the McDonaldization of society.

  1. A customer cleaning their table and disposing of waste after eating at a restaurant. This would make the process of serving a customer more efficient. By having customers clear their own table, that is less time and resources the McDonald’s has to spend on the customer.
  2. A doctor seeing a patient at his or her residence rather than at a medical facility. This is the opposite of answer choice A because this would decrease efficiency for the doctor. Because we have that decrease in efficiency, this is NOT a representation of the McDonaldization of society and is a good answer. The doctor would rather see patients quickly in their medical facility instead. 
  3. A supermarket chain using self-scan machines at check-outs, in place of employees. This has to do with control. By using self-scan machines, the supermarket becomes more efficient and predictable. This is a representation of the McDonaldization of society so we can eliminate this answer choice.
  4. A chain of coffee shops offering the same menu and décor across the United States. This has to do with predictability and standardization. We expect the same service at any McDonald’s chain, and the coffee shop offering the same menu and décor is achieving that same predictability and standardization. This is a representation of the McDonaldization of society so we can eliminate this answer choice. We can stick with answer choice B as the best answer choice.